Algebra learning

Polinomi, disuguaglianze, numeri complessi, ...
scambret
Messaggi: 734
Iscritto il: 23 mag 2012, 20:49
Località: Acquarica del Capo

Re: Algebra learning

Messaggio da scambret »

Sono contento che tutti i problemi sono andati via! Penultima sessione dei problemi!

Hint sui problemi 17
Testo nascosto:
17.1. (IMOSL 2007 A3) L’idea è maggiorizzare la frazione con una potenza di $x$. Con $\displaystyle \frac{1+x^{2k}}{1+x^{4k}} < \frac{1}{x^{k}}$

17.2. (IMO 2009/5) Dimostrare che $f(1)=1$ altrimenti $f$ è periodica. Ora $f$ è biettiva e si dimostra per induzione che $f(n)=(n-1)f(2)-(n-2)$

17.3. (IMOSL 2011 A3) Con $P(x,0)$, $P(0,x)$ si ottiene $g$ in funzione di $f$. Con $P(x+y,0)$ si ottiene $f(x+y)$ in funzione di $f(y)$. Da qui si pone $y=c$.
18.1. Sia $n \geq 3$ un intero e $a_2, a_3, \cdots a_n$ reali positivi tali che $a_2 a_3 \cdots a_n =1$. Dimostrare che

$$(1+a_2)^2(1+a_3)^3 \cdots (1+a_n)^n > n^n$$

18.2. Trovare tutte le funzioni $f: \mathbb{R}^+ \to \mathbb{R}^+$ tali che per ogni $x, y \in \mathbb{R}^+$ vale

$$f(x+f(y)) = f(x+y)+f(y)$$

18.3. Trovare tutte le funzioni $f: \mathbb{Q}^+ \to \mathbb{Q}^+$ tali che per ogni $x, y \in \mathbb{Q}^+$ vale

$$f(f(x)^2y)=x^3f(xy)$$
TheRoS
Messaggi: 30
Iscritto il: 25 feb 2018, 13:05

Re: Algebra learning

Messaggio da TheRoS »

Provo il 18.1
Per ogni $i$ appartenente a $(2,\dots,n)$ compiamo il seguente ragionamento.
\begin{equation}
a_i+1=a_i+\frac{1}{i-1}+\dots+\frac{1}{i-1}
\end{equation}
Dove $\frac{1}{i-1}$ compare $i-1$ volte. Applichiamo ora $AM-GM$ su questi $i$ termini attendo che:
\begin{equation}
\frac{a_i+1}{i}\geq(\frac{a_i}{(i-1)^{i-1}})^{1/i}\iff (a_i+1)^i\geq\frac{i^i\cdot a_i}{(i-1)^{i-1}}
\end{equation}
Grazie a questa osservazione possiamo scrivere che:
\begin{equation}
(a_2+1)^2\cdot\dots\cdot(a_n+1)^n\geq \frac{4a_2}{1}\cdot\dots\cdot\frac{i^i\cdot a_i}{(i-1)^{i-1}}\cdot\dots\cdot\frac{n^n\cdot a_n}{(n-1)^{n-1}}=n^n
\end{equation}
scambret
Messaggi: 734
Iscritto il: 23 mag 2012, 20:49
Località: Acquarica del Capo

Re: Algebra learning

Messaggio da scambret »

Sei molto vicino per chiudere la soluzione!
TheRoS
Messaggi: 30
Iscritto il: 25 feb 2018, 13:05

Re: Algebra learning

Messaggio da TheRoS »

Ah giusto, c'è da dimostrare che l'uguaglianza non va bene.
Si ha l'uguaglianza quando ogni $a_i=\frac{1}{i-1}$, ma se ciò fosse vero, il prodotto di questi viene minore di 1 e perciò si esclude.
Parmenide
Messaggi: 27
Iscritto il: 30 mag 2018, 21:24

Re: Algebra learning

Messaggio da Parmenide »

Provo il 18.3:
Testo nascosto:
Dimostro che l'unica soluzione è $\displaystyle{f(x)=\frac{1}{x}}$, che sostituendo verifica.
Sia $P(x;y)$ l'equazione funzionale del testo

Dimostro per prima cosa che $f$ è iniettiva:
da $P(x;1)$ si ottiene $f\left(f\left(x^2\right)\right)=x^3f\left(x\right)$

da cui, se $f(x)=f(y)$, si ha $\displaystyle{x^3=\frac{f\left(f\left(x^2\right)\right)}{f(x)}=\frac{f\left(f\left( y^2\right)\right)}{f(y)}=y^3 \Rightarrow x=y }$, quindi $f$ è iniettiva.


Dimostro ora che $f$ è moltiplicativa:
da $P(xy;1)$, $P(x;y)$, $P(y;f(x)^2)$ si ottiene $\displaystyle{f\left(f\left(xy\right)^2\right)=y^3x^3f\left(xy\right)=y^3f\left(f(x)^2y\right)=f\left(f(x)^2f(y)^2\right)}$
da cui, per l'iniettività, $f(xy)^2=f(x)^2f(y)^2 \Rightarrow f(xy)=f(x)f(y)$, quindi $f$ è moltiplicativa


A questo punto chiaramente si ha che $f(1)=1$ e $f\left(x^n\right)=f\left(x\right)^n$

In questo modo posso riscrivere $P(x;y)$ come $f\left(f(x)\right)^2f(y)=x^3f(x)f(y) \Rightarrow f\left(f(x)\right)=\displaystyle{\sqrt{x^3f(x)}}$

Sia ora $g(x)=xf(x)$.
Si ha $g(g(x))=g(xf(x))=xf(x)f(xf(x))=xf\left(x^2f(x)\right)=xf\left(x^2\right)f\left(f(x)\right)=\displaystyle{xf(x)\sqrt{x^3f(x)}}=\left[xf(x)\right]^{\frac{5}{2}}=\left[g(x)\right]^{\frac{5}{2}}$

e per induzione $g$ composto se stesso $n+1$ volte è $\left[g(x)\right]^{\left(\frac{5}{2}\right)^n}$

A questo punto fissiamo $x$ nell'equazione $g \circ g\circ … \circ g=\left[g(x)\right]^{\left(\frac{5}{2}\right)^n}$ dove nel LHS la composizione è ripetuta $n+1$ volte.

Dimostro ora che $g(x)=1$:
LHS è sempre razionale per come sono definite $f,g$, quindi anche $\left[g(x)\right]^{\left(\frac{5}{2}\right)^n}$ deve essere sempre razionale.
Supponiamo per assurdo $g(x)\neq 1$ e sia la scomposizione in fattori primi di $g(x)=p_1^{\alpha_1}...p_k^{\alpha_k}$ dove i $p_i$ sono primi distinti e gli $\alpha_i$ sono interi diversi da 0.
Allora la fattorizzazione di $g\circ g\circ …\circ g=p_1^{\left(\frac{5}{2}\right)^n \alpha_1}...p_k^{\left(\frac{5}{2}\right)^n \alpha_k}$
Quindi tutti gli esponenti dovrebbero essere sempre interi, ma ad esempio il primo esponente non lo è se $2^n \nmid \alpha_1$, contraddizione.

Allora $g(x)=1$, che porta a $\displaystyle{f(x)=\frac{1}{x}}$

scambret
Messaggi: 734
Iscritto il: 23 mag 2012, 20:49
Località: Acquarica del Capo

Re: Algebra learning

Messaggio da scambret »

Ultima sessione di questo mini progetto. Un in bocca al lupo agli IMOisti di quest'annata e per gli altri, arrivederci!

Hint sui problemi 18
Testo nascosto:
18.1. (IMO 2012/2) Una AM-GM pesata per far uscire solo un $a_k$ e gli altri $k-1$ termini devono essere uguali e con somma 1.

Altrimenti si faceva anche con un’idea di analisi: “Trovare il più grande $m$ tale che $(1+x)^k \geq mx$”. Derivando si ottiene che $f’(x)=0$ per $\displaystyle x_0=\left(\frac{m}{k}\right)^{\frac{1}{k-1}}$. Ora però deve essere anche vero che $f(x_0)=0$ da cui $\displaystyle m=\frac{k^k}{(k-1)^{k-1}}$. Ora moltiplicando tutto viene.

18.2. (IMOSL 2007 A4) Quando avete $\mathbb{R}^+$, l’idea è fare delle sostituzioni vietate per dimostrare alcune caratteristiche di $f$. Qui si dimostra che $f(x)>x$, allora $g(x)=f(x)-x$ che sarà iniettiva e da qui bisogna dimostrare che è additiva e crescente.

18.3. (IMOSL 2010 A5) $f$ iniettiva. Per sfruttarla a pieno, dopo $P(x,1)$ vorrei qualcosa come $x^3f(x)$ e dunque pongo $P(x,f^2(y))$. Ora posso porre $g(x)=xf(x)$.
19.1. Trovare tutte le funzioni $f: \mathbb{R} \to \mathbb{R}$ tali che per ogni $x, y$ reali vale

$$f(f(x)f(y))+f(x+y) = f(xy)$$

19.2. Trovare tutte le funzioni $f: \mathbb{R} \to \mathbb{R}$ tali che per ogni $x, y$ reali vale

$$f(x+f(x+y)) + f(xy) = x + f(x+y) + yf(x)$$

19.3. Sia $f: \mathbb{Q}^+ \to \mathbb{R}$ una funzione che soddisfa le tre condizioni

a) Per ogni $x, y \in \mathbb{Q}^+$ vale $f(x)f(y) \geq f(xy)$
b) Per ogni $x, y \in \mathbb{Q}^+$ vale $f(x+y) \geq f(x)+f(y)$
c) Esiste un razionale $a>1$ tale che $f(a)=a$

Dimostrare che $f(x)=x$ per ogni $x \in \mathbb{Q}^+$
Avatar utente
Pit
Messaggi: 23
Iscritto il: 22 ago 2017, 10:22

Re: Algebra learning

Messaggio da Pit »

Mi ero dimenticato di questo post.

19.1. (feat. Giorgia Benassi e Sabrina Botticchio)
Testo nascosto:
Le funzioni possibili sono $0,x-1$ e $1-x$ che chiaramente soddisfano, mostriamo che sono le uniche.
Sia $P(x,y)$ l'equazione del testo.
$f$ è soluzione se $-f$ lo è, WLOG $f(0)\leq 0$.
Supponiamo che $f$ non sia identicamente nulla.
Da $P(0,0)$ si ottiene $f(f(0)^2)=0$, quindi esiste almeno un reale che annulla la funzione.
Se esiste un reale $t$ tale che $f(t)=0$ e $t\neq 1$, da $P(t,\frac{t}{t-1})$ si ottiene $$f(0)=0$$
Da $P(x,0)$ si ottiene $f(x)=0$ assurdo $\Rightarrow f(t)=0$ se e solo se $t=1\Rightarrow f(0)^2=1\Rightarrow f(0)=-1$
Da $P(x,1)$ si ottiene $$f(x+1)=f(x)+1$$ e induttivamente $f(x+n)=f(x)+n$
Supponiamo per assurdo che esistano due reali $u,v$ distinti tali che $f(u)=f(v)$.
Prendiamo $n$ tale che $(v+n)^2-4(u+n-1)\geq 0$, quindi esistono $x,y$ tali che $xy=u+n-1$ e $x+y=v+n$, sostituendoli in $P$ si ottiene $$f(f(x)f(y))+f(v+n)=f(u+n-1)\Rightarrow f(f(x)f(y))+f(v)+n=f(u)+n-1\Rightarrow f(f(x)f(y)+1)=0\Rightarrow f(x)f(y)=0$$ WLOG $f(y)=0\Rightarrow y=1\Rightarrow u+n-1=x=v+n-1\Rightarrow u=v$ assurdo $\Rightarrow f$ è iniettiva.
Da $P(x,-x)$ si ottiene $$f(f(x)f(-x))=f(-x^2)+1=f(1-x^2)\Rightarrow f(x)f(-x)=1-x^2$$
Da $P(1-x,x)$ si ottiene $$f(f(x)f(1-x))=f(x-x^2)\Rightarrow f(x)f(1-x)=x-x^2\Rightarrow x-x^2=f(x)+f(x)f(-x)=1-x^2\Rightarrow f(x)=x-1$$
Da questo le tre soluzioni.

19.3.
Testo nascosto:
Sia $P(x,y)$ la prima disuguaglianza e $S(x,y)$ la seconda.
Da $P(1,a)$ si ottiene $$af(1)\geq a\Rightarrow f(1)\geq 1$$ Da $S(x,1)$ si ottiene induttivamente $f(n)\geq n$.
Da $P\left(\frac{n}{m},m\right)$ si ottiene $$f\left(\frac{n}{m}\right)f(m)\geq f(n)\geq n>0\Rightarrow f\left(\frac{n}{m}\right)>0$$
con $m,n$ interi positivi. Quindi per ogni razionale $x$ si ha $f(x)>0$.
Per ogni $x$ non intero, da $S(x-\lfloor x\rfloor,\lfloor x\rfloor)$ si ottiene $$f(x)\geq f(x-\lfloor x\rfloor)+f(\lfloor x\rfloor)\geq f(\lfloor x\rfloor)\geq \lfloor x\rfloor>x-1$$ Da $P(x,y)$ otteniamo induttivamente $f(x)^n\geq f(x^n)$ e quindi $$f(x)^n> x^n-1\Rightarrow f(x)>\sqrt[n]{x^n-1}$$
Per $x>1$ si ottiene $$\lim\limits_{n\to\infty}\sqrt[n]{x^n-1}=e^{\lim\limits_{n\to\infty}\frac{\ln(x^n-1)}{n}}$$ dato che numeratore e denominatore del limite tendono entambi ad infinito, per il teorema di de l'hopital si ottiene $$\lim\limits_{n\to\infty}\sqrt[n]{x^n-1}=e^{\lim\limits_{n\to\infty}\frac{x^n\ln(x)}{x^n-1}}=e^{\ln(x)}=x\Rightarrow f(x)\geq x$$ per x>1.
Da $f(x)^n\geq f(x^n)$ si ottiene $$a^n= f(a)^n\geq f(a^n)\geq a^n\Rightarrow f(a^n)=a^n$$
Fissiamo $x$ maggiore di 1 e prendiamo $n$ tale che $a^n>x+1$, da $S(a^n-x,x)$ otteniamo $$a^n=f(a^n)=f((a^n-x)+x)\geq f(a^n-x)+f(x)\geq a^n-x+x=a^n$$$\Rightarrow f(x)=x$ per ogni $x>1$
Da $S(x,y)$ otteniamo induttivamente $$f(nx)\geq nf(x)$$ Fissato $x$ e prendendo $n$ intero sufficientemente grande si ottiene $$nx=f(nx)\geq nf(x)\Rightarrow f(x)\leq x$$ Da $P(x,n)$ si ottiene $$f(x)f(n)\geq f(nx)\Rightarrow nf(x)\geq f(nx)=nx\Rightarrow f(x)\geq x$$ E quindi $f(x)=x$ per ogni $x$ razionale, che è la tesi.
Nessuno :?:
Lello01
Messaggi: 27
Iscritto il: 08 feb 2018, 13:37
Località: Malo

Re: Algebra learning

Messaggio da Lello01 »

19.2
Testo nascosto:
Sia p(x,y) :f(x+(x+y) )+f(xy)=x+f(x+y)+yf(x)
Le due soluzioni sono f(x)=x e f(x)=2-x
Che verificano in quanto x+x+y+xy=x+x+y+xy
E f(2-y)+2-xy=2-y+2y-xy che vale in quando y+2-xy=2+y-xy.
p(0,y):f(f(y) )+f(0)=f(y)+yf(0)
Ora ho due casi: o f(0)=0 o f(0)≠0
-se f(0)=0 allora p(x,0) dà f(x+f(x) )=x+f(x) (2)
p(x-1,1) dà f(x-1+f(x) )=x-1+f(x) (3)
Riscriviamo ora p(x,y) come f(x+(x+y) )-x-yf(x)=f(x+y)-f(xy) da cui simmetrizzando
f(x+f(x+y) )-x-yf(x)=f(y+f(x+y))-y-xf(y)
Poniamo ora y=-x da cui f(x)-x+xf(x)=f(-x)+x-xf(-x) da cui sistemando viene
(x+1)f(x)-2x=f(-x)(1-x)∀x (1)
Se pongo x=1 in (1) ottengo f(1)=1 mentre se pongo x=-1 ottengo f(1)=-1.
Definiamo ora x+f(x)=A
Per (2) F(A)=A mentre per (3) F(A-1)=A-1
P(1,A-1) ora dà f(1+A)+f(A-1)=1+A+A-1→F(1+A)=1+A
Quindi riprendendo la definizione di A si ha che f(x+f(x)+1)=x+f(x)+1
Ora P(x+1;-1) dà f(x+f(x)+1)+f(-x-1)=x+1+f(x)-f(x+1)→f(-x-1)=-f(x+1)
Da cui ∀x f(x)=-f(-x)
Ma allora sostituendo ciò in (1) (x+1)f(x)-2x=f(x)(x-1)→2f(x)=2x→f(x)=x
-f(0)≠0
Allora p(x-1,1) dà f(x-1+f(x) )=x-1+f(x)
Quindi f ha almeno un punto fisso ;consideriamo ora uno dei punti fissi che chiameremo z. Ma allora
P(0,z): dà f(f(z) )+f(0)=f(z)+zf(0)→zf(0)=f(0)→z=1
Quindi l’unico punto fisso di f(x) è 1 ma in quanto f(x-1+f(x) )=x-1+f(x) si deve avere che
x-1+f(x)=1→f(x)=2-x
Avatar utente
Pit
Messaggi: 23
Iscritto il: 22 ago 2017, 10:22

Re: Algebra learning

Messaggio da Pit »

Dopo poco più di un anno volevo riproporre questa serie di problemi molto istruttivi per condividere con voi un bellissimo viaggio verso la materia olimpica (indiscutibilmente) più bella.

Colgo l'occasione per aggiungere le soluzioni mancanti (sperando di non aver sbagliato niente) e per ringraziare scambret per averli proposti.

1.1 Talete
Testo nascosto:
Fissata una circonferenza $Ω$ di raggio $R$, come posso scegliere tre punti $A$, $B$ e $C$ su $Ω$ di modo che l'area del triangolo $ABC$ sia la massima possibile? Se fisso $A$ e $B$, il massimo è quando $C$ sta sull'asse di $AB$, e così ciclicamente. Quindi il massimo possibile è quando $ABC$ è equilatero. L'area del triangolo equilatero inscritto in una circonferenza di raggio $R$ è
$$\frac{3\sqrt{3}}{4}⋅R^2.$$

Dunque per un qualsiasi triangolo $ABC$, detta $S$ la sua area e $R$ il raggio della sua circonferenza circoscritta, si ha
$$S\leq\frac{3\sqrt{3}}{4}⋅R^2.$$

Eleviamo il tutto al quadrato e moltiplichiamo per 16: si ottiene
$$16⋅S^2≤27⋅R^4.$$

Siano ora $x$, $y$ e $z$ i tre lati di $ABC$. È piuttosto noto che
$$xyz=4RS,$$

e dunque in particolare anche
$$256⋅S^4=\frac{x^4y^4z^4}{R^4}.$$

Moltiplicando membro a membro questa con la disuguaglianza ottenuta prima, si ottiene
$$4096⋅S^6≤27⋅x^4y^4z^4.$$

Dividendo per $4096$ ed estraendo la radice cubica, si ottiene
$$S^2≤\frac{3}{16}⋅(xyz)^{4/3}.$$

Questa vale per ogni terna $x$, $y$, $z$ di reali positivi che sono i lati di un triangolo, quindi in particolare vale per $x=a+b$, $y=b+c$ e $z=c+a$. Ricordando che $S^2=abc(a+b+c)$, ho dimostrato la tesi.
1.2 Pit
Testo nascosto:
Siano $S=a+b+c$, $Q=ab+ac+bc$ e $P=abc=1$.
Per $AM-GM$ su $(a^2b^2,a^2c^2)$ si ha $a^2b^2+a^2c^2\geq 2a^2bc$, sommando le versioni cicliche si ottiene $$a^2b^2+a^2c^2+b^2c^2\geq a^2bc+ab^2c+abc^2$$$$\Rightarrow a^2b^2+a^2c^2+b^2c^2+2(a^2bc+ab^2c+abc^2)\geq 3(a^2bc+ab^2c+abc^2)$$ $$\Rightarrow Q^2\geq 3SP=3S$$
Per $AM-GM$ su $\left(\frac{Q}{3},\frac{Q}{3},\frac{3}{S}\right)$ si ottiene $$\frac{2Q}{3}+\frac{3}{S}\geq 3\sqrt[3]{\frac{Q^2}{3S}}\geq 3$$
Per $AM-GM$ su $(ab+ac+bc)$ si ha $$\frac{Q}{3}\geq 1$$
Infine sommando le ultime due si ottiene $$Q+\frac{3}{S}\geq 4\Rightarrow SQ+3\geq 4S$$$$SQ-1\geq 4(S-1)\Rightarrow SQ-P\geq 4(S-1)$$$$\Rightarrow (a+b)(a+c)(b+c)\geq 4(a+b+c-1)$$ che è la tesi.
1.3 Pit
Testo nascosto:
Siano $x=\frac{a}{b}$, $y=\frac{b}{c}$, $z=\frac{c}{a}$, $S=x+y+z$, $Q=xy+xz+yz$, $P=xyz=1$ e $t=\sqrt{SQ}.$
Per $AM-GM$ su $(x,y,z)$ e $(xy,xz,yz)$ si ottiene $S\geq 3$ e $Q\geq 3\Rightarrow t\geq 3$
Riscriviamo il RHS della disuguaglianza come $$abc+\sqrt[3]{(a^3+abc)(b^3+abc)(c^3+abc)}=abc+abc\sqrt[3]{(x+y)(x+z)(y+z)}=$$$$=abc\left(1+\sqrt[3]{SQ-P}\right)=abc\left(1+\sqrt[3]{t^2-1}\right)$$ mentre il LHS come $$\sqrt{(a^2b+b^2c+ac^2)(ab^2+bc^2+a^2c)}=abc\sqrt{(x+y+z)\left(\frac{1}{x}+\frac{1}{y}+\frac{1}{z}\right)}=$$$$=abc\sqrt{(x+y+z)\left(\frac{xy+xz+yz}{xyz}\right)}=abc\sqrt{\frac{SQ}{P}}=abct$$
Quindi la disuguaglianza è vera $$\iff t\geq 1+\sqrt[3]{t^2-1}$$ ma dato che $t\geq 3$, questa è equivalente a $$(t-1)^3\geq t^2-1\iff t^3-4t^2+3t\geq 0$$ $$\iff t(t-3)(t-1)\geq 0$$ che è chiaramente vera per la condizione trovata su $t$.

2.1 Pit
Testo nascosto:
Notiamo prima di tutto che il coefficiente direttivo è positivo (se non lo fosse, $P(x)$ sarebbe negativo per $x$ arbitrariamente grande).

Dimostriamo prima la tesi nel caso in cui $P$ non abbia radici reali, dato che $z$ è una radice complessa di $P$ se e solo se lo è anche $\bar{z}$, siano $a_1+ib_1,a_2+ib_2,...,a_k+ib_k,a_1−ib_1,a_2−ib_2,...,a_k−ib_k$ le radici di $P$ e $m>0$ il coefficiente direttivo $$⇒P(x)=m∏\limits_{j=0}^{j=k}(x−(a_j+ib_j))(x−(a_j−ib_j)).$$ Chiamando $R(x)+iS(x)=∏\limits_{j=0}^{j=k}(x−(a_j+ib_j))$ con $R$ e $S$ polinomi a coefficienti reali, si ha $$∏\limits_{j=0}^{j=k}(x−(a_j−ib_j))=R(x)−iS(x)⇒P(x)=m(R(x)+iS(x))(R(x)−iS(x))=(\sqrt{m}R(x))^2+(\sqrt{m}S(x))^2.$$
Passiamo ora al caso generale, ogni radice reale di $P$ ha molteplicità pari (altrimenti se $α$ fosse una radice con molteplicità dispari, $P(α+ϵ)$ e $P(α−ϵ)$ per $ϵ$ arbitrariamente piccolo avrebbero segni diversi) $⇒P(x)$ si può scrivere come $$m∏(x−α_j)^2∏(x−(a_j+ib_j))(x−(a_j−ib_j))$$ da cui come prima si ottiene $$P(x)=(\sqrt{m}R(x)∏(x−α_j))^2+(\sqrt{m}S(x)∏(x−α_j))^2.$$ Che è la tesi.
2.2 Pit e sab01
Testo nascosto:
Dimostriamo la tesi nel caso in cui esistono infiniti numeri interi positivi che rendono $P$ un quadrato (da questo deriva il caso generale dato che in caso contrario basterebbe considerare $P(-x)$).
Sia $P(x)=x^{2k}+a_{2k-1}x^{2k-1}+...+a_1x+a_0$. Consideriamo il sistema in $k$ equazioni nelle variabili $(b_0,b_1,...,b_{k-1})$ $$\begin{cases}
2b_{k-1} &= a_{2k-1} \\ b^2_{k-1}+2b_{k-2} &= a_{2k-2} \\ 2b_{k-1}b_{k-2}+2b_{k-3}&=a_{2k-3}\\ b^2_{k-2}+2b_{k-1}b_{k-3}+2b_{k-4}&=a_{2k-4}\\...
\end{cases}$$ ovvero quello ottenuto uguagliando i coefficienti di grado maggiore di $k-1$ di $P(x)$ e $Q(x)^2$ dove $Q(x)=x^k+b_{k-1}x^{k-1}+...+b_1x+b_0$.
Il sistema ha soluzione con $b_0,b_1,...,b_{k-1}$ razionali dato che, risolvendo le equazioni dall'alto verso il basso, ad ogni equazione compare una sola variabile nuova e moltiplicata per $2$.
Supponiamo per assurdo $P(x)\neq Q(x)^2$.
Sia $d>0$ il minimo comune multiplo dei denominatori dei coefficienti di $Q$, allora anche $d^2P(x)$ è un quadrato per infiniti interi positivi.
Sia inoltre $R(x)=d^2P(x)-d^2Q(x)^2$. Questo è di grado minore di $k$ per come abbiamo scelto $Q$.
Sia $n$ un intero positivo arbitrariamente grande tale che $d^2P(n)$ è un quadrato.
Se $d^2P(n)>d^2Q(n)^2$ otteniamo $$d^2P(n)\geq (dQ(n)+1)^2=d^2Q(n)^2+2dQ(n)+1$$ $$\Rightarrow 2dQ(n)-R(n)+1\leq 0$$ ma $2dQ(n)-R(n)+1$ è un polinomio di grado $k$ con coefficiente direttore positivo $(2d)$ e quindi per $n$ arbitrariamente grande è positivo.
Similmente se $d^2P(n)<d^2Q(n)^2$ si ha $$d^2P(n)\leq (dQ(n)-1)^2=d^2Q(n)^2-2dQ(n)+1$$ $$\Rightarrow 2dQ(n)+R(n)-1\leq 0$$ che come prima è assurdo per $n$ arbitrariamente grande.
Quindi $d^2P(n)=d^2Q(n)^2$ per infiniti $n\Rightarrow P(x)=Q(x)^2$.
Rimane da dimostrare che $Q$ è a coefficienti interi, supponiamo per assurdo che non lo sia.
Dal sistema iniziale abbiamo induttivamente che tutti i denominatori dei coefficienti di $Q$ ridotti ai minimi termini sono potenze di $2$. Sia $2^j$ la potenza più alta tra queste e supponiamo che sia il denominatore di $b_i\Rightarrow b_i=\frac{l}{2^j}$ con $l$ intero dispari. Confrontanto i coefficienti di grado $2i$ di $P(x)=Q(x)^2$ otteniamo $$b_{i}^2+2A=a_{2i}$$ dove $A$ è un termine formato da somme di prodotti di coppie di coefficienti di $Q(x)$ e quindi anche $A$ ha termini con denominatori solo potenze di due e sono tutti minori o uguali a $2^j$. Moltiplichiamo i termini di questa relazione per $2^{2j-1}$ e otteniamo $$2^{2j-1}b_i^2=2^{2j-1}a_{2i}-2^{2j}A$$ $$\Rightarrow 2^{2j-1}b_i^2\in \mathbb{Z}\Rightarrow \frac{l}{2}\in\mathbb{Z}$$ $\Rightarrow l$ è pari, assurdo. Quindi $Q$ è a coefficienti interi, che è la tesi.
2.3 Pit
Testo nascosto:
Lemma $1$: Sia $Q(x)$ un polinomio non costante a coefficienti interi e $k\geq 1$ un intero. Allora l'insieme dei primi che dividono almeno uno tra $Q(k),Q(k+1),Q(k+2),...$ è infinito.
Dimostrazione: Sia $Q(x)=\sum\limits_{i=0}^n a_ix^i$. Se $a_0=0$ allora $p|Q(p)$, consideriamo ora $a_0\neq 0$ e supponiamo per assurdo che l'insieme dei primi sia finito e sia $P$ il prodotto di questi. Allora $$Q(hPa^2_0)=a_0(a_nh^nP^na_0^{2n-1}+...+a_1hPa_0+1)$$ ma per ogni $h$ intero positivo tale che $h\geq\frac{k}{Pa_0^2}$, il termine tra parentesi non può avere fattori primi essendo coprimo con tutti quelli che dividono $Q(n)\Rightarrow R(x)=a_nP^na_0^{2n-1}x^n+...+a_1Pa_0x+1$ vale infinite volte almeno uno tra $1$ e $-1\Rightarrow $ è costante e quindi anche $Q$ lo è, assurdo.


Lemma $2$: Sia $Q(x)$ un polinomio non costante a coefficienti interi tale che $Q(n)$ è il quadrato di un intero per ogni $k$ intero $\geq 1$. Allora $Q(x)$ è il quadrato di un polinomio a coefficienti interi.
Dimostrazione : Sia $Q(x)=R(x)^2G(x)$ con $R$ e $G$ a coefficienti interi e $G$ prodotto di fattori irriducibili distinti $\Rightarrow G(x)$ è libero da quadrati.
$Q(n)$ è un quadrato $\iff G(n)$ lo è.
Dato che per il lemma $1$ esistono infiniti primi che dividono un numero della forma $G(n)$ con $n\geq k$, sappiamo che esistono primi $p$ arbitrariamente grandi tali che $p^2|G(x)$ (essendo $G(n)$ un quadrato).
Essendo $a_i(n+p)^i\equiv a_in^i+pia_ix^{i-1}$ modulo $p^2$, otteniamo $$G(n+p)\equiv_{p^2} G(n)+pG'(n)$$ $$\Rightarrow p|G(n+p)\Rightarrow p^2|G(n+p)$$ $$p^2|pG'(n)\Rightarrow p|G'(n)$$ perciò esistono primi arbitrariamente grandi per i quali esiste un intero $n_p$ per cui $p|(G(n_p),G'(n_p))$.
Per il lemma di Bézout esistono due polinomi a coefficienti razionali $A(x),B(x)$ tali che $$A(x)G(x)+B(x)G'(x)=(G(x),G'(x))$$ Supponiamo per assurdo che $(G(x),G'(x))=c$ sia una costante non nulla e sia $d$ il minimo comune multiplo di tutti i denominatori di $A(x)$ e $B(x)$ $$\Rightarrow p|dA(n_p)G(n_p)+dB(n_p)G'(n_p)=cd$$ $\Rightarrow cd$ è divisibile per primi arbitrariamente grandi, assurdo. Quindi $(G(x),G'(x))$ non è una costante non nulla e quindi $G(x)$ non è libero da quadrati, da cui la tesi.


Passiamo ora al problema.
Supponiamo prima che $P$ non sia costante e abbia grado $n\geq 1$.
Il polinomio $P(x^2-1)+P(1)$ è un quadrato per ogni intero $x\geq 2\Rightarrow$ per il lemma $2$ otteniamo $P(x^2-1)+P(1)=T(x)^2$ per un qualche $T$ a coefficienti interi. Confrontando i coefficienti di grado massimo nelle due espressioni otteniamo che il coefficiente direttore di $P$ deve essere un quadrato, chiamiamolo $l^2$.
Prendiamo ora $a=16x^2$ e $b=9x^2$, per il lemma $2$ otteniamo $$P(16x^2)+P(9x^2)=R(x)^2$$ per un qualche $R$ a coefficienti interi. Sia $c$ il coefficiente direttore di $R(x)$, confontando i coefficienti dei termini di grado massimo otteniamo $$l^2(4^{2n}+3^{2n})=c^2\Rightarrow l|c\Rightarrow c=kl$$ $$\Rightarrow \left(4^n\right)^2+\left(3^n\right)^2=k^2$$ $\Rightarrow \left(4^n,3^n,k\right)$ è una terna pitagorica, è primitiva dato che $3$ e $4$ sono coprimi e quindi esistono $u>v$ coprimi e di parità diversa tali che $$4^n=2uv$$ e $$3^n=u^2-v^2=(u-v)(u+v)$$ da quest'ultima si ha $u-v=3^\alpha$ e $u+v=3^\beta$ con $\alpha<\beta$ interi e la cui somma è $n$; dalla prima invece si ha che $u$ e $v$ sono potenze di $2$.
Se $v\geq 1$, avremmo $3^\beta\equiv_2 0$, assurdo $\Rightarrow v=1$ $$\Rightarrow 3^\beta=3^\alpha+2$$ ma le uniche potenze di $3$ che distano $2$ sono $3^1$ e $3^0 \Rightarrow n=\alpha+\beta=1$.
Sia $P(x)=a^2x+t$ $$\Rightarrow T(x)^2=a^2x^2-a^2+t+a^2+t=a^2x^2+2t$$
con $t\neq 0$ avremmo un assurdo visto che nello sviluppo di $T(x)^2$ ci sarebbe anche il termine di primo grado $\Rightarrow P(x)=a^2x$ e questo soddisfa per ogni $a$ intero.
Rimane il caso in cui $P$ sia costante, sia $P(x)=f$. La condizione del testo diventa $2f$ è un quadrato $\Rightarrow P(x)=2c^2$ che è soluzione per ogni $c$ intero.

3.1 Pit
Testo nascosto:
$WLOG$ $y$ è compreso tra $x$ e $z$, quindi $$z(y−z)(y−x)≤0⇒y^2z+xz^2≤xyz+yz^2≤2xyz+yz^2$$ $$⇒x^2y+y^2z+xz^2≤x^2y+2xyz+yz^2≤y(x+z)^2=y(1−y)^2$$
Ma per AM-GM sulla terna $\left(\frac{1−y}{2},\frac{1−y}{2},y\right)$, si ottiene $$\frac{1}{3}=\frac{\frac{1-y}{2}+\frac{1-y}{2}+y}{3}\geq \sqrt[3]{\frac{y(1-y)^2}{4}}\Rightarrow x^2y+y^2z+xz^2≤\frac{4}{27}$$ che è la tesi.
3.2 Pit
Testo nascosto:
Mostriamo che vale la disuguaglianza $$\sum_{cyc} \frac{1}{\left( x+y \right)^2} \geq \frac{9}{4(xy+xz+yz)}$$ per ogni $x,y,z\geq 0$
$WLOG\;x\geq y\geq z$.
Sia $$f(x,y,z)=\sum_{cyc} \frac{1}{\left( x+y \right)^2} -\frac{9}{4(xy+xz+yz)}$$
Sia inoltre $t=\sqrt{xy}$, allora abbiamo $$f(x,y,z)-f(t,t,z)=$$ $$\left(\frac{1}{\left( x+y \right)^2}-\frac{1}{4xy}\right)+\left(\frac{1}{\left( x+z \right)^2}+\frac{1}{\left(y+z \right)^2}\right)-\frac{2}{\left( t+z \right)^2}+\left(\frac{9}{4(xy+2zt)}-\frac{9}{4(xy+xz+yz)}\right)=$$ $$=\frac{-(x-y)^2}{4xy(x+y)^2}+\frac{2(x+z)(y+z)+(x-y)^2}{(x+z)^2(y+z)^2}-\frac{2}{\left( t+z \right)^2}+\frac{9z\left(\sqrt{x}-\sqrt{y}\right)^2}{4(xy+xz+yz)(xy+2zt)}=$$ $$=\left(\frac{-(x-y)^2}{4xy(x+y)^2}+\frac{(x-y)^2}{(x+z)^2(y+z)^2}\right)+\left(\frac{2}{(x+z)(y+z)}-\frac{2}{\left( t+z \right)^2}\right)+\frac{9z\left(\sqrt{x}-\sqrt{y}\right)^2}{4(xy+xz+yz)(xy+2zt)}=$$ $$=\left(\frac{(x-y)^2}{(x+z)^2(y+z)^2}-\frac{(x-y)^2}{4xy(x+y)^2}\right)+\left(\frac{9z\left(\sqrt{x}-\sqrt{y}\right)^2}{4(xy+xz+yz)(xy+2zt)}-\frac{2z\left(\sqrt{x}-\sqrt{y}\right)^2}{(x+z)(y+z)(t+z)^2}\right)$$ la prima parentesi è non negativa dato che $4xy\geq y^2+z^2+2yz=(y+z)^2$ e $(x+y)^2\geq (x+z)^2$.
Per la seconda invece vale $$9(x+z)(y+z)(t+z)^2-8(xy+xz+yz)(xy+2zt)=$$ $$=x^2y^2+9xyz^2+2xyzt+x^2yz+9xz^3+2xz^2t+xy^2z+9yz^3+2yz^2t\geq 0$$
da cui $$f(x,y,z)\geq f(t,t,z)$$
Infine abbiamo $$f(t,t,z)=\frac{1}{4t^2}+\frac{2}{\left( t+z \right)^2}-\frac{9}{4t(t+2z)}=\frac{z(t-z)^2}{2t^2(t+z)^2(t+2z)}\geq 0$$ $$\Rightarrow f(x,y,z)\geq f(t,t,z)\geq 0$$ da cui la tesi.
3.3 Pit
Testo nascosto:
$WLOG \; z=\min (x,y,z)$.
Sia $$f(x,y,z)=\sum_{cyc} \frac{1}{\left( x-y \right)^2} - \frac{4}{xy+yz+xz}$$ Allora abbiamo $$f(x,y,z)-f(x-z,y-z,0)=$$$$=\frac{1}{\left( x-y \right)^2}+\frac{1}{\left( x-z \right)^2}+\frac{1}{\left( y-z \right)^2}-\frac{4}{xy+yz+xz}-\frac{1}{\left( x-y \right)^2}-\frac{1}{\left( x-z \right)^2}-\frac{1}{\left( y-z \right)^2}+\frac{4}{(x-z)(y-z)}=$$ $$4\cdot\frac{(xy+xz+yz)-(x-z)(y-z)}{(x-z)(x-y)(xy-xz+yz)}=\frac{4z(2x+2y-z)}{(x-z)(x-y)(xy-xz+yz)}\geq 0$$ posti quindi $a=x-z$ e $b=y-z$, si ha $$f(x,y,z)\geq f(a,b,0)$$ Infine abbiamo $$f(a,b,0)=\frac{1}{(a-b)^2}+\frac{1}{a^2}+\frac{1}{b^2}-\frac{4}{ab}=$$ $$\frac{a^2b^2+b^2(a-b)^2+a^2(a-b)^2-4ab(a-b)^2}{a^2b^2(a-b)^2}=\frac{11a^2b^2+a^4b^4-6a^3b-6ab^3}{a^2b^2(a-b)^2}$$$$\frac{(a^2+b^2-3ab)^2}{a^2b^2(a-b)^2}\geq 0\Rightarrow f(x,y,z)\geq f(a,b,0)\geq 0$$ da cui la tesi.

4.1 Sirio
Testo nascosto:
Dividendo i due membri dell'uguaglianza data otteniamo:
$$f(x)+f(y)=\frac{f(x^2-y^2)}{x-y}$$
Per ogni $x$,$y$ reali distinti.
Essendo il primo membro dell'uguaglianza appena scritta simmetrico in $x,y$, lo è anche il secondo. Per ogni $x,y$ reali distinti, vale quindi la seguente uguaglianza:
$$\frac{f(x^2-y^2)}{x-y}=\frac{f(y^2-x^2)}{y-x}$$
Moltiplicando i due membri per $x−y$ otteniamo che, per ogni $x,y$ reali distinti, vale:
$$f(x^2-y^2)=-f(y^2-x^2)$$
Per $x,y$ reali distinti, la quantità $z:=x^2−y^2$ può assumere qualunque valore reale non nullo. Infatti, fissato un valore reale non nullo per $z$ ed un valore reale per $x$ con $x^2>z$, si ha che $y=\sqrt{x^2−z}$ soddisfa la relazione che definisce $z$. Si ha quindi, per ogni $z$ reale non nullo, la seguente uguaglianza:
$f(z)=−f(−z)$
In realtà, questa uguaglianza vale anche per $z=0$ poiché, come si può verificare sostituendo $x=0,y=0$ nell'uguaglianza data dal testo, si ha $f(0)=0$.
Bene, ricordando $f(z)=−f(−z)$ e l'uguaglianza del testo, otteniamo che le seguenti uguaglianze sono vere per ogni $x,y$ reali:
$$(x−y)(f(x)+f(y))=f(x^2−y^2)=f(x^2−(−y)^2)=(x+y)(f(x)−f(y))$$
$$xf(y)−yf(x)=−xf(y)+yf(x)$$
$$yf(x)=xf(y)$$
Sostituendo $y=1$ e definendo $a:=f(1)$, si ha, per ogni $x$ reale:
$f(x)=ax$
Sostituendo le soluzioni di questa forma nel testo otteniamo un'identità. Le soluzioni sono quindi tutte e sole quelle di questa forma.
4.2 Pit
Testo nascosto:
Sia $P(x,y)$ l'equazione del testo. Supponiamo per assurdo che esistano due reali positivi $a$ e $b$ tali che $f(a)=f(b)$.
Confrontando $P(a,x)$ e $P(b,x)$ otteniamo che $$a^2(b+x)=b^2(a+x)$$ per ogni reale positivo $x$, quindi $$ab(a-b)+(a+b)(a-b)x=0\Rightarrow 0=ab+x(a+b)>0$$ assurdo, quindi la funzione è iniettiva.
Da $P\left(\frac{3}{2},\frac{3}{4}\right)$ si ha $$f\left(f\left(\frac{3}{2}\right)\cdot\frac{3}{4}\right)=f\left(f\left(\frac{3}{2}\right)+f\left(\frac{3}{4}\right)\right)$$$$\Rightarrow 3f\left(\frac{3}{2}\right)=4f\left(\frac{3}{2}\right)+4f\left(\frac{3}{4}\right)$$ $$\Rightarrow 0=f\left(\frac{3}{2}\right)+4f\left(\frac{3}{4}\right)>0$$ che è un assurdo, quindi non esistono funzioni che rispettano l'equazione del testo.
4.3 Davide Di Vora
Testo nascosto:
Sia $P(x;y)$ l'equazione funzionale del testo.
Fissando $x$ otteniamo che il $RHS$ può variare su tutto $\mathbb{R}$ e quindi $f$ è surgettiva.
Siano ora $a$ e $b$ due reali tali che $f(a)=f(b)$, allora da $P(1;a)$ e $P(1;b)$ ottengo
$$a+f(1)=f(f(a)+1)=f(f(b)+1)=b+f(1)$$

e quindi $a=b$, da cui segue $f$ iniettiva.
Visto che $f$ è bigettiva esiste un unico valore reale $h$ tale che $f(h)=0$; da $P(h;h)$ ottengo $h=0.$
Da $P(1;−f(1))$ sfruttando l'iniettività ottengo
$$f(f(−f(1))+1)=0=f(0)$$

e quindi $f(−f(1))=−1$
Da $P\left(x;−\frac{f(x)}{x}\right)$ ottengo
$$f\left(xf\left(-\frac{f(x)}{x}\right)+x\right)=0 \iff f\left(-\frac{f(x)}{x}\right)=-1=f(-f(1))$$

e quindi per l'iniettività
$$f(x)=kx$$

Sostituendo nel testo si verifica che le uniche soluzioni sono $f(x)=x$ e $f(x)=−x$.

5.1 Roob
Testo nascosto:
Sia $P(a,b,c)$ l'equazione del testo. Da $P(a,0,0)$ otteniamo
$$f(f(a)+2f(0))=a$$
per cui $f$ è iniettiva e suriettiva. Considerando $P(a,b,0)$ e $P(a+b,0,0)$ otteniamo che
$$f(f(a)+f(b)+f(0))=a+b=f(f(a+b)+2f(0))$$
quindi, per l'iniettività
$$f(a)+f(b)+f(0)=f(a+b)+2f(0)⟺f(a)+f(b)=f(a+b)+f(0)$$
Ponendo $g(x)=f(x)−f(0)$ abbiamo che
$$g(a)+g(b)+2f(0)=g(a+b)+2f(0)⟺g(a)+g(b)=g(a+b)$$
e visto che $g$ ha $N$ come dominio, $g(x)=cx$ per qualche $c$ naturale, e quindi $f(x)=cx+d$, con $c$ e $d$ naturali.
Sostituendo nell'equazione originale, e ponendo $b=c=0$, otteniamo che $c^2a+3cd+d=a$. Se avessimo $c>1$ allora avremmo $c^2a+3cd+d≥c^2a>a$, e visto che $c=0$ è impossibile (avremmo a uguale a una costante $∀a∈N$, assurdo) $c=1$. Di conseguenza dobbiamo avere $d=0$.
Poichè essa soddisfa, $f(x)=x$ è l'unica soluzione.
5.2 Davide Di Vora
Testo nascosto:
Sia $P(x;y;z)$ la disuguaglianza funzionale del testo.
Da P$(x;0;0)$ ottengo
$$f(0)≥f(x)$$

Da $P(x;x;−x)$ ottengo
$$f(2x)≥f(0)$$

e quindi
$$f(0)≥f(x)≥f(0)$$

Da cui $f(x)=f(0)$ che sostituendo si verifica che è soluzione.
5.3 Pit
Testo nascosto:
Sia $P(x,y)$ la funzionale. Da $P(1,x)$ si ha
$$f(f(x)+f(1))=2f(1)+x$$
da cui $f(x)$ è bigettiva. Sia $a$ tale che $f(a)=0$, da $P(a,a)$ si ha
$$f(0)=a^2$$

Da $P(a,0)$ si ha
$$f(a^3)=0=f(a)⇒a^3=a⇒a(a^2−1)=0$$

Se $a=0$, da $P(x,0)$, ponendo $f(x)=z$, ottengo
$$f(f(x))=2f(x)⇒f(z)=2z$$
che però non soddisfa l'equazione iniziale. Si deve avere quindi $a^2=1$. Da $f(0,y)$, usando $f(0)=a^2=1$, si ha
$$f(1)=2⇒a=−1$$

Da $P(1,x)$ si ha
$$f(f(x)+2)=x+4$$
e sia $Q(x)$ quest'ultima equazione. Da $Q(−2)$ si ha
$$f(f(−2)+2)=2=f(1)⇒f(−2)+2=1⇒f(−2)=−1$$
Da $Q(−4)$ si ha
$$f(f(−4)+2)=0=f(−1)⇒f(−4)+2=−1⇒f(−4)=−3$$

Prendiamo $b$ tale che $f(b)=2b$, da $P(b,−4)$ si ha
$$f(−b)=0=f(−1)$$

Quindi $f(t)=2t$ se e solo se $t=1$. Da $P(x,−2)$ si ha
$$f(f(x)−x)=2(f(x)−x)$$
che per quanto visto prima, vale se e solo se $f(x)−x=1⇒f(x)=x+1$
Sostituendo quest'ultima nell'equazione iniziale si vede che verifica, infatti
$$f(xf(y)+f(x))=f(xy+2x+1)=xy+2(x+1)=2f(x)+xy$$

6.1 Pit
Testo nascosto:
Troviamo prima le soluzioni costanti. $$f(x)\equiv c\Rightarrow c^2=c$$ quindi abbiamo le soluzioni $f(x)=0$ e $f(x)=1$.
Supponiamo ora che $f(x)$ non sia costante.
Sia $g(x)=f(x^2)-f(x)f(x-1)$. Questo polinomio è nullo per ogni $x$ reale, quindi è identicamente nullo $\Rightarrow f(x^2)=f(x)f(x-1)$ per ogni $x$ complesso.
Dimostriamo prima che se $f$ ha una radice diversa da $0,1$ e $-1$ allora è identicamente nullo.
Sia $a$ tale radice, ponendo $x\mapsto a$, otteniamo che $a^2$ è radice. Ponendo induttivamente $x\mapsto a^{2^k}$ otteniamo che tutti i numeri di questa forma (che sono infiniti per come abbiamo scelto $a$) sono radici di $f\Rightarrow f$ è nullo.
Essendo $f$ un polinomio non costante, ha almeno una radice complessa . Per quanto appena dimostrato dimostrato, le radici possono essere solo $0,1$ e $-1$.
Se $1$ fosse radice $x\mapsto 2\Rightarrow f(4)=0$ assurdo.
Se $0$ fosse radice $x\mapsto 1\Rightarrow f(1)=0\Rightarrow f(4)=0$ assurdo.
Se infine $-1$ fosse radice $x\mapsto 0\Rightarrow f(0)=0\Rightarrow f(1)=0\Rightarrow f(4)=0$ assurdo.
Quindi le uniche soluzioni sono le due costanti.
6.2 FedeX333X
Testo nascosto:
Osserviamo innanzitutto che $x^2−x+1$ e $x^2+x+1$ non hanno radici reali. Poiché $x^2+x+1$ non divide $x^2−x+1$ e viceversa, dobbiamo necessariamente avere che $x^2+x+1∣g(x^2+x+1)$ e $x^2−x+1∣f(x^2−x+1)$; abbiamo quindi che $f(x^2−x+1)=(x^2−x+1)⋅F(x^2−x+1)$ per un altro opportuno polinomio $F(x)$, cioè che $f(x)=xF(x)$, ad allo stesso modo $g(x)=xG(x)$ per un opportuno polinomio $G(x)$. Sostituendo nell'equazione iniziale, ricaviamo che $F(x^2−x+1)=G(x^2+x+1)$. Poniamo $x\mapsto −(x+1)$. Abbiamo $F(x^2+3x+3)=G(x^2+x+1)$, ma essendo $G(x^2+x+1)=F(x^2−x+1)$, ricaviamo $F(x^2+3x+3)=F(x^2−x+1)$. Vogliamo ora dimostrare che $F(x)$ è costante. Osserviamo che possiamo scrivere $x^2−x+1$ come $\left(x−\frac{1}{2}\right)^2+\frac{3}{4}$ e $x^2+3x+3$ come $\left(x+\frac{3}{2}\right)^2+\frac{3}{4}$. Per quanto appena trovato abbiamo che $F\left[\left(x-\frac{1}{2}\right)^2+\frac{3}{4}\right]=F\left[\left(x+\frac{3}{2}\right)^2+\frac{3}{4}\right]$; ma allora, se $p(x)=F\left[\left(x+\frac{3}{2}\right)^2+\frac{3}{4}\right]$, $p(x)=p(x−2)∀x∈R.$ Quindi $p(x)$ è un polinomio periodico, e quindi costante; ma allora anche $F(x)$ e $G(x)$ sono costanti, cioè $F(x)=G(x)=k$. Sostituendo nell'equazione iniziale, osserviamo che effettivamente $f(x)=g(x)=kx$ la soddisfa, e questi sono tutti e soli i tali polinomi che soddisfano le ipotesi.
6.3 Pit
Testo nascosto:
Le soluzioni sono $f(x)=\alpha x^4 +\beta x^2$. Prima di tutto verifichiamo che queste funzionano, sostituiamo nell'equazione (e incrociamo le dita) $$\alpha ((a-b)^4+(a-c)^4+(b-c)^4-2(a+b+c)^4)+\beta ((a-b)^2+(a-c)^2+(b-c)^2-2(a+b+c)^2)$$
Il primo termine addendo è uguale a $$\alpha(2a^2(ab+ac+bc)+2a^2bc+2b^2(ab+ac+bc)+2ab^2c+2c^2(ab+ac+bc)+2abc^2+a^2b^2+a^2c^2+b^2c^2)=$$$$=\alpha(a^2b^2+a^2c^2+b^2c^2+2a^2bc+2ab^2c+2abc^2)=\alpha(ab+ac+bc)^2=0$$ mentre il secondo termine è uguale a $$-6\beta(ab+ac+bc)=0$$ quindi verifica, mostriamo ora che sono le uniche soluzioni.
Poniamo $a=b=c=0$ e otteniamo $f(0)=0$.
Da $b\mapsto 0$, $c\mapsto 0$ si ha $f(a)=f(-a)$ quindi $f$ è pari.
Ponendo $a\mapsto 6x$, $b\mapsto 3x$ e $c\mapsto -2x$ (questi si possono prendere perché $ab+ac+bc=18x^2-12x^2-6x^2=0$ e usando la parità di $f$ si ottiene $$f(3x)+f(5x)+f(8x)=2f(7x)$$ questa deve essere un'uguaglianza valida per ogni $x$ e quindi devono coincidere tutti i coefficienti tra il membro di sinistra e di destra.
Sia $n\geq 1$ il grado di $f$.
Confrontando il termine di grado $n$ si ottiene $$2\cdot 7^n=3^n+5^n+8^n>8^n\Rightarrow \left(\frac{8}{7}\right)^n< 2$$ se $n\geq 6$ si ha (usando la crescenza del termine di sinistra) $$2>\left(\frac{8}{7}\right)^n\geq \left(\frac{8}{7}\right)^6>2$$ assurdo $\Rightarrow n<6$ e per la parità di $f$ e usando $f(0)=0$, compaiono solo termini di grado pari $\Rightarrow f(x)=\alpha x^4+\beta x^2$ che sono quindi le uniche soluzioni.

7.1 Linda_
Testo nascosto:
Se abbiamo $a,b,c∈R^+$ sicuramente ce ne sono 2 tra questi entrambi $≥1$ oppure $≤1$. Senza perdita di generalità (la disuguaglianza è simmetrica) diciamo che sono $b,c$. Quindi $(1−b)(1−c)≥0$.
$$a^2+b^2+c^2+2abc+1−2(ab+bc+ca)=a^2−2a+1+2a+b^2−2bc+c^2+2abc−2ab−2ca$$$$=(a−1)^2+(b−c)^2+2a(1−c)−2ab(1−c)$$$$=(a−1)2+(b−c)^2+2a(1−b)(1−c)$$

e poiché $(a−1)^2+(b−c)^2+2a(1−b)(1−c)$ è somma di non negativi (ricordiamo che $a>0$ e $(1−b)(1−c)≥0$) allora $(a−1)^2+(b−c)^2+2a(1−b)(1−c)≥0$, quindi $a^2+b^2+c^2+2abc+1−2(ab+bc+ca)≥0$ da cui la tesi
$$a^2+b^2+c^2+2abc+1≥2(ab+bc+ca)$$
7.2 Linda_
Testo nascosto:
Dimostriamo che $0≤ab+bc+ca−abc$.
Sicuramente almeno uno tra $a,b,c$ dev'essere $<1$: se così non fosse avremmo $a^2+b^2+c^2+2abc≥5>4$, assurdo. wlog diciamo $a<1$.
$ab+cb+ca−abc=ab+ca+bc(1−a)$ ed essendo $a,b,c>0$ e $1−a>0$ allora $ab+bc+ca−abc=ab+ca+bc(1−a)≥0$

Dimostriamo ora che $ab+bc+ca−abc≤2$.
Possiamo dire che 2 tra $a,b,c$ (diciamo $a,b$, tanto tutto è simmetrico) sono entrambi $≥1$ o entrambi $≤1$, quindi $(1−a)(1−b)≥0$ da cui $a+b−ab≤1$.
$ab+bc+ca−abc=ab+c(a+b−ab)≤ab+c$
Ora se mostriamo che $ab+c≤2$ concludiamo. Supponiamo che sia $ab+c>2$. Allora
$$a^2+b^2+c^2+2abc=a^2+b^2+c(c+ab)+abc>2ab+2c+abc>4+abc>4$$
assurdo perché per ipotesi $a^2+b^2+c^2+2abc=4$.
Quindi $ab+c≤2$, da cui $ab+bc+ca−abc≤2$
7.3 Pit
Testo nascosto:
Ponendo $a=b=c=\sqrt{2}$, otteniamo $36k\leq 27\Rightarrow k\leq\frac{3}{4}$. Mostriamo ora che vale la disuguaglianza $$4(1+a^2)(1+b^2)(1+c^2) \geq 3(ab+bc+ca)^2$$ per concludere che $k=\frac{3}{4}$ è la costante migliore.
Tra i numeri $a^2,$ $b^2$ e $c^2$ almeno due sono entrambi $\geq$ o $\leq$ di $2$. $WLOG$ sono $b^2$ e $c^2$, quindi si ha $$(b^2-2)(c^2-2)\geq 0\Rightarrow b^2c^2+4 \geq 2b^2+2c^2$$
Per $CS$ sulla coppia di vettori $(b+c,bc)$, $(a,1)$ si ottiene $$(ab+ac+bc)^2=(a(b+c)+bc)^2\leq ((b+c)^2+b^2c^2)(a^2+1)$$
quindi $$3(ab+bc+ca)^2\leq 3((b+c)^2+b^2c^2)(a^2+1)$$ Ci basta quindi mostrare che $$4(1+b^2)(1+c^2)\geq 3((b+c)^2+b^2c^2)$$
Questa è vera se e solo se $$4(1+b^2)(1+c^2)- 3((b+c)^2+b^2c^2)\geq 0$$ $$\Leftrightarrow 4b^2c^2+4b^2+4c^2+4-3b^2-3c^2-6bc-3b^2c^2\geq 0$$ $$\Leftrightarrow b^2c^2+4 +b^2+c^2-6bc\geq 0$$ usando la disuguaglianza di prima, ci basta mostrare che $$2b^2+2c^2+b^2+c^2-6bc\geq 0\Leftrightarrow 3(b-c)^2\geq 0$$ che è chiaramente vera.

8.1 Pit
Testo nascosto:
Riscriviamo la disuguaglianza del testo come $$f(x)-f(x+y)\geq \frac{f(x)y}{f(x)+y}$$ e chiamiamola $P(x,y)$.
Dati due reali positivi $a>b$, da $P(b,a-b)$ si ottiene $$f(b)-f(a)\geq \frac{f(b)(a-b)}{f(b)+a-b}>0\Rightarrow f(a)<f(b)$$ quindi la funzione è decrescente. Fissiamo $x$ reale positivo e scegliamo $n$ intero positivo tale che $n>\frac{1}{f(x+1)}$.
Per ogni $k$ intero tale che $0\leq k\leq n-1$ si ha (usando la decrescenza di $f$) $$\frac{\frac{1}{n}+f\left(x+\frac{k}{n}\right)}{\frac{1}{n}\cdot f\left(x+\frac{k}{n}\right)}=n+\frac{1}{f\left(x+\frac{k}{n}\right)}\leq n+\frac{1}{f(x+1)}\leq 2n$$$$\Rightarrow\frac{\frac{1}{n}\cdot f\left(x+\frac{k}{n}\right)}{\frac{1}{n}+ f\left(x+\frac{k}{n}\right)}\geq \frac{1}{2n}$$
Da quest'ultima disuguaglianza e da $P\left(x+\frac{k}{n},\frac{1}{n}\right)$ si ottiene $$f\left(x+\frac{k}{n}\right)-f\left(x+\frac{k+1}{n}\right)\geq\frac{\frac{1}{n}\cdot f\left(x+\frac{k}{n}\right)}{\frac{1}{n}+ f\left(x+\frac{k}{n}\right)}\geq \frac{1}{2n} $$ $$\Rightarrow f\left(x+\frac{k}{n}\right)-f\left(x+\frac{k+1}{n}\right)\geq\frac{1}{2n}$$ sommando quest'ultima per ogni $0\leq k\leq n-1$ si ha $$f(x)-f(x+1)\geq \frac{1}{2}$$
Infine dato $m$ un intero positivo si ottiene $$f(x)-f(x+m)=(f(x)-f(x+1))+(f(x+1)-f(x+2))+...+(f(x+m-1)-f(x+m))\geq\frac{m}{2}$$ $$f(x)\geq f(x+m)+\frac{m}{2}>\frac{m}{2}$$
per ogni $x$, ma questa è chiaramente falsa per $m$ arbitrariamente grande $\Rightarrow$ nessuna funzione soddisfa la disequazione del testo.
8.2 Pit
Testo nascosto:
Siano $P(x,y)$ la disequazione del testo, $a=f(0)>0$ e $c=f(a)$.
Da $P(0,c)$ si ha $$f(c)\geq a+c^2>c^2$$
Per $x$ sufficientemente grande, da $P(a,x-a)$ si ottiene $$f(x)\geq c+(x-a)f(c)=c+xf(c)-af(c)>c-af(c)+c^2x>\frac{c^2x}{2}$$
quindi per $x$ sufficientemente grande esiste un reale positivo $b$ tale che $f(x)>bx$.
Da $P(x,f(x)-x)$ si ha $$f(f(x))\geq f(x)+(f(x)-x)f(f(x))$$ $$\Rightarrow f(f(x))(1+x-f(x))\geq f(x)$$
ma per $x$ sufficientemente grande $f(x)>bx>0$ e $f(f(x))>bf(x)>b^2x$ $$\Rightarrow f(f(x))(1+x-f(x))\geq f(x)>0\Rightarrow 1+x-f(x)>0$$ $$\Rightarrow f(x)<x+1$$ usando quest'ultima e $f(x)>bx$ in $P(x,x)$ (sempre per $x$ arbitrariamente grande) si ottiene $$1+2x>f(2x)\geq f(x)+xf(f(x))>bx+b^2x^2$$ $$\Rightarrow b^2x^2+x(b-2)-1<0$$ che non vale per $x$ arbitrariamente grande, assurdo $\Rightarrow$ non esistono funzioni che rispettano le condizioni del testo.
8.3 Pit
Testo nascosto:
Sia $P(x,y)$ la disequazione del testo.
Da $P(0,0)$ si ha $f(0)^2+2f(0)\leq 0\Rightarrow -2\leq f(0)\leq 0$.
Sia $M=\sup_{x \neq 0} |f(x)|$ e $\{x_i\}_{i\geq 0}$ una sequenza di reali non nulli tali che $\lim\limits_{n\to\infty} |f(x_n)| = M$.
Da $P\left(x_n,\frac{x}{x_n}\right)$ con $x\neq 0$ si ottiene $$M^2 \geq f\left(x_n + \frac{x}{x_n}\right)^2 \geq 2f(x) + f(x_n)^2 + f\left(\frac{x}{x_n}\right)^2 \geq 2f(x) + [f(x_n)]^2 $$ e facendo il limite per $n\to\infty$ si ha $$ M^2\geq 2f(x) + M^2\Rightarrow f(x)\leq 0$$ per ogni $x\neq 0$, ma l'abbiamo dimostrata precedentemente anche per $x=0$ e quindi vale per ogni $x$ reale.
Siano $g(x)=-f(x)\geq 0$, $m=\sup_{x \neq 0} g(x)$, $\{y_i\}_{i\geq 0}$ una sequenza di reali non nulli tali che $\lim\limits_{n\to\infty} g(y_n) = m$, $P(x,y)$ diventa $$g(x+y)^2\geq g(x)^2-2g(xy)+g(y)^2$$
Da $P\left(y_n,y_n\right)$ si ottiene $$m^2\geq g\left(2y_n\right)^2 \geq -2g(y_n^2) + 2g(y_n)^2\geq 2g(y_n)^2-2m$$ e facendo il limite per $n\to\infty$ si ha $$m^2\geq 2m^2-2m\Rightarrow m^2\leq 2m\Rightarrow 0\leq m\leq 2$$$$g(x)\leq 2\Rightarrow f(x)\geq -2$$ $$\Rightarrow -2\leq f(x)\leq 0$$ da cui la tesi.

9.1 PG93
Testo nascosto:
Si verifica facilmente che $f(a,b,c)=ab+c+1+bc+a+1+ca+b+1+(1−a)(1−b)(1−c)≤1$ definita per $a,b,c∈[0,1]$ è convessa in ognuna delle tre variabili. Perciò il massimo di $f$ è ottenuto per $a,b,c∈\{0;1\}$, cioè agli estremi del dominio di $f$. Verifichiamo che per queste otto terne si ha $f(a,b,c)=1$, e ciò permette di concludere.
9.2 Pit
Testo nascosto:
Supponiamo per assurdo che gli $a_i$ siano tutti minori di $2$. Poniamo quindi $x_i=2-a_i>0$ per ogni $1\leq i\leq n$.
Siano $$S=x_1 + x_2 + \cdots + x_n\textrm{ e } S_2=x_1^2+ x_2^2 + \cdots + x_n^2$$
Nell'espansione di $S^2$ è presente $S_2$ sommato a termini positivi $\Rightarrow S_2<S^2$
Dalle condizioni del testo otteniamo $$n\leq a_1 + a_2 + \cdots + a_n=2n-S\Rightarrow S\leq n $$$$n^2\leq a_1^2+ a_2^2 + \cdots + a_n^2=4n+S_2-4S< 4n+S^2-4S$$$$\Rightarrow S^2-4S+4n-n^2>0\Rightarrow (S-n)(S+n-4)>0$$
ma il secondo fattore è positivo mentre il primo è $\leq 0$, assurdo. Da cui la tesi.
9.3 Pit
Testo nascosto:
Chiamiamo $y_i=\frac{x_i}{2017}>0$ e $z_i=\frac{1}{yi+1}>0$, da cui $y_i=\frac{1}{z_i}−1$. La condizione diventa
$$\frac{1}{y_1+1}+\frac{1}{y_2+1}+...+\frac{1}{y_n+1}=z_1+z_2+...+z_n=1$$

mentre la tesi è equivalente a
$$\sqrt[n]{y_1\cdots y_n}\geq n-1$$

Consideriamo la funzione $f(x)=\ln\left(\frac{1}{x}-1\right)$. Questa è convessa in $\left(0,\frac{1}{2}\right]$ e quindi se $0<z_i≤\frac{1}{2}$ per ogni $i$ da 1 a $n$, per Jensen si ha
$$\frac{f(z_1)+f(z_2)+...+f(x_n)}{n}\geq f\left(\frac{z_1+z_2+...+z_n}{n}\right)=f\left(\frac{1}{n}\right)\Rightarrow$$
$$\frac{1}{n}\left[\ln\left(\frac{1}{z_1}-1\right)+\ln\left(\frac{1}{z_2}-1\right)+...+\ln\left(\frac{1}{z_n}-1\right)\right]\geq \ln(n-1)\Rightarrow$$
$$\frac{1}{n}\left[\ln(y_1y_2\cdots y_n)\right]\geq \ln(n-1)\Rightarrow \ln\left(\sqrt[n]{y_1y_2\cdots y_n}\right)\geq\ln(n-1)\Rightarrow \sqrt[n]{y_1y_2\cdots y_n}\geq n-1$$

Se invece per almeno un $j$ si ha $z_j>\frac{1}{2}$, supponiamo $j=1$. Chiaramente questo è unico dato che se ce ne fosse almeno un altro si avrebbe $\sum\limits_{k=1}^n z_k>1$ che va contro la condizione.
Chiamiamo $a=\frac{z_1+z_2}{2}\leq\frac{1}{2}$. Vale la disuguaglianza
$$\left(\frac{1}{a}-1\right)^2\leq \left(\frac{1}{z_1}-1\right)\left(\frac{1}{z_2}-1\right)=y_1y_2$$
(se non ho sbagliato i calcoli) questa dovrebbe essere equivalente a $(z_1−z_2)^2(1−z_1−z_2)≥0$. Per Jensen sulla $n$-upla $(a,a,z_3,...,z_n)$ si ottiene similmente a prima
$$n-1\leq \sqrt[n]{\left(\frac{1}{a}-1\right)^2y_3y_4\cdots y_n}\leq\sqrt[n]{y_1y_2\cdots y_n}$$
che è la tesi.
Ultima modifica di Pit il 07 nov 2019, 21:40, modificato 1 volta in totale.
Nessuno :?:
Avatar utente
Pit
Messaggi: 23
Iscritto il: 22 ago 2017, 10:22

Re: Algebra learning

Messaggio da Pit »

10.1 Sirio
Testo nascosto:
Trattiamo prima il caso $n=0$ e poi quello $n>0$.

Sostituendo $n=0$ nell'equazione del testo otteniamo:
$\left(f(0)\right)^3+\left(g(0)\right)^3+\left(h(0)\right)^3=0$
Da cui, ricordando che $f(0)$ e cicliche sono naturali, quindi sono maggiori o uguali a zero, quindi la somma dei loro cubi è maggiore o uguale a zero, otteniamo che affinché quest'ultima somma sia nulla è necessario $f(0)=g(0)=h(0)=0$ $(1)$.

Supponiamo ora $n>0$. Per la $(1)$ e per l'ipotesi che le tre funzioni sono biunivoche e con immagine naturale abbiamo $f(n)>0$ e cicliche.
Dividiamo per tre i due membri dell'equazione del testo ed estraiamone quindi la radice cubica:
$\displaystyle \sqrt[3]{\dfrac{\left(f(n)\right)^3+\left(g(n)\right)^3+\left(h(n)\right)^3}3}=\sqrt[3]{n·g(n)·h(n)}\;\;\;\forall n\in\mathbb N*$ $(2)$
Applicando ora CM-GM otteniamo:
$\displaystyle \sqrt[3]{\dfrac{\left(f(n)\right)^3+\left(g(n)\right)^3+\left(h(n)\right)^3}3}\geq\sqrt[3]{f(n)·g(n)·h(n)}\;\;\;\forall n\in\mathbb N*$
Da cui, per la $(2)$:
$\sqrt[3]{n·g(n)·h(n)}\geq\sqrt[3]{f(n)·g(n)·h(n)}\;\;\;\forall n\in\mathbb N*$
Da cui:
$n≥f(n)\;\;\;\forall n\in\mathbb N*$
Sia il minimo $m>0$ tale che $m>f(m)=a$. Essendo $a<m$, si ha $f(a)=a=f(m)\Rightarrow a=m$, assurdo $\Rightarrow f(n)=n$.
Tornando alla $(2)$, sostituendo $f(n)=n$ otteniamo un'uguaglianza tra media cubica e media geometrica, da cui segue $g(n)=h(n)=n\;\;\;∀n∈N∗$.

Quindi, ricordando anche la $(1)$, abbiamo che $f,g,h$ sono tutte e tre l'identità.
10.2 Pit
Testo nascosto:
Mostriamo per induzione su $m\geq -1$ che per $n>m$ si ha $f(n)>m$.
Il passo base è banalmente vero, quindi passiamo a quello induttivo.
Prendiamo $n>m+1\Rightarrow n-1>m$ per ipotesi induttiva $$f(n-1)>m\Rightarrow f(f(n-1))>m\Rightarrow f(f(n-1))\geq m+1$$
Ponendo nella disequazione iniziale $n\mapsto n-1$ si ottiene $f(n)>f(f(n-1))\geq m+1$ che completa l'induzione.
Prendendo $m=n-1$ si ottiene $f(n)\geq n$, usando questa nella disequazione del testo si ha $f(n+1)>f(f(n))\geq f(n)\Rightarrow f$ è strettamente crescente. Da questo si ha (visto che $f(n+1)>f(f(n))$) $n+1>f(n)$ e questo conclude dato che $$n+1>f(n)\geq n\Rightarrow f(n)=n$$ e questa chiaramente soddisfa la condizione del testo.
10.3 Pit
Testo nascosto:
Essendo $\mathbb{N}$ il codominio della funzione, questa ammette minimo. Sia $c$ questo minimo e $a$ un intero tale che $f(a)=c$.
Ponendo $k\mapsto a-3$ si ha $$6c=6f(a)=3f(a-1)+2f(a-2)+f(a-3)\geq 6c$$ quindi deve valere l'uguaglianza $\Rightarrow f(a-1)=f(a-2)=f(a-3)=c$.
mostriamo ora che per induzione estesa decrescente che per ogni $x\leq a$ si ha $f(x)=c$.
Come passi base abbiamo già mostrato che $f(a)=f(a-1)=f(a-2)=f(a-3)=c$, passiamo a quello induttivo.
Supponiamo che $f(n)=c$ per ogni $a\geq n\geq x$, poniamo $k\mapsto x-1$ e otteniamo $$f(x-1)=6f(x+2)-3f(x+1)-2f(x)=c$$
che completa questa induzione.
Mostriamo ora sempre per induzione estesa che $f(x)=c$ per ogni $x$ intero, abbiamo mostrato già la tesi per $x\leq a$.
Supponiamo quindi che $f(n)=c$ per ogni $n\leq x$, poniamo $k\mapsto x-2$ e otteniamo $$6f(x+1)=3f(x)+2f(x-1)+f(x-2)=6c\Rightarrow f(x+1)=c$$ che completa questa seconda induzione $\Rightarrow f$ è costante.
Infine chiaramente qualsiasi funzione costante rispetta l'equazione e quindi queste sono tutte e sole le soluzioni.

11.0 Pit
Testo nascosto:
1) Lemma: siano $\textbf{a}=(a_1,a_2,...,a_n)$ e $\textbf{b}=(b_1,b_2,...,b_n)$ due vettori ordinati allo stesso modo e con tutte le componenti non negative. Per ogni permutazione $\sigma: \{1,2,...,n\}\mapsto \{1,2,...,n\}$ si ha $$(a_1 + b_1) \cdots (a_n+b_n) \leq (a_1 + b_{\sigma(1)}) \cdots (a_n + b_{\sigma(n)})$$
Dimostrazione:
$WLOG$ i vettori sono ordinati in modo crescente. Consideriamo una permutazione $\sigma$ diversa dall'identità e prendiamo $i$ e $j$ con $i<j$ tali che $\sigma(i)>\sigma(j)$ e sia $P=\prod\limits_{k\neq i,\;k\neq j} (a_k+b_{\sigma(k)})$. Allora avremo $$P (a_i+b_{\sigma(i)}) (a_j+b_{\sigma(j)})-P (a_i+b_{\sigma(j)}) (a_j+b_{\sigma(i)})=$$ $$=P(a_ib_{\sigma(j)}+a_jb_{\sigma(i)}-a_ib_{\sigma(i)}-a_jb_{\sigma(j)})=P(a_j-a_i)(b_{\sigma(i)}-b_{\sigma(j)})\geq 0$$ $$\Rightarrow P (a_i+b_{\sigma(i)}) (a_j+b_{\sigma(j)})\geq P (a_i+b_{\sigma(j)}) (a_j+b_{\sigma(i)})$$ allora considerando la permutazione $\sigma_1(j)=\begin{cases} \sigma(i), & \mbox{se }k\mbox{ =j} \\ \sigma(j), & \mbox{se }k\mbox{ =i} \\ \sigma(k), & \mbox{altrimenti}
\end{cases}$ si ha $$(a_1 + b_{\sigma_1(1)}) \cdots (a_n + b_{\sigma_1(n)})\leq (a_1 + b_{\sigma(1)}) \cdots (a_n + b_{\sigma(n)})$$ ma data qualsiasi permutazione iniziale possiamo effettuare questo scambio di due elementi un numero finito di volte per ottenere l'identità (usando il selection Sort) e rendendo il relativo prodotto minore o uguale a quello iniziale, da cui la tesi.

2) Con dimostrazione analoga (invertendo i segni) si ha che se i due vettori sono ordinati in maniera opposta, allora $$(a_1 + b_1) \cdots (a_n+b_n) \geq (a_1 + b_{\sigma(1)}) \cdots (a_n + b_{\sigma(n)})$$
11.1 Pit
Testo nascosto:
Siano $b_1,b_2,...,b_n$ i numeri $a_1,a_2,...,a_n$ permutati in ordine crescente.
I vettori $(b_1,b_2,...,b_n)$ e $\left(2+\frac{1}{b_1},2+\frac{1}{b_2},...,2+\frac{1}{b_n}\right)$ sono ordinati in maniera opposta dato che $f(x)=2+\frac{1}{x}$ è decrescente.
Allora per il secondo lemma abbiamo (essendo gli $a_i$ una permutazione di $b_i$) $$\left( \frac{1}{a_1} + a_2 + 2 \right) \left( \frac{1}{a_2} + a_3 + 2 \right) \cdots \left( \frac{1}{a_n} + a_1 + 2 \right)\leq \left( \frac{1}{b_1} + b_1 + 2 \right) \left( \frac{1}{b_2} + b_2 + 2 \right) \cdots \left( \frac{1}{b_n} + b_n + 2 \right)=$$ $$=\left( \frac{1}{a_1} + a_1 + 2 \right) \left( \frac{1}{a_2} + a_2 + 2 \right) \cdots \left( \frac{1}{a_n} + a_n + 2 \right)=\frac{(a_1+1)^2(a_2+1)^2 \cdots (a_n+1)^2}{a_1a_2 \cdots a_n}$$ che è la tesi.
11.2 Pit
Testo nascosto:
Essendo $g(x)=x^3+1$ crescente, per il primo lemma si ha $$\prod_{cyc} (a_i^3+a_{i+1}+1) \geq\prod_{cyc} (a_i^3+a_{i}+1) $$
Dato che $f(x)=\frac{1}{x}+\frac{1}{x^2}$ è decrescente, per il secondo lemma si ottiene $$\prod_{cyc} \left(a_i+\frac{1}{a_i}+\frac{1}{a_i^2}\right)\geq\prod_{cyc} \left(a_i+\frac{1}{a_{i+1}}+\frac{1}{a_{i+1}^2}\right) $$ $$\Rightarrow \prod_{cyc}a_i^2\prod_{cyc} \left(a_i+\frac{1}{a_i}+\frac{1}{a_i^2}\right)\geq\prod_{cyc}a_{i+1}^2\prod_{cyc} \left(a_i+\frac{1}{a_{i+1}}+\frac{1}{a_{i+1}^2}\right)$$ $$\Rightarrow \prod_{cyc} (a_i^3+a_{i}+1)\geq \prod_{cyc} (a_ia_{i+1}^2+a_{i+1}+1)$$ e unendo le due disuguaglianze si ha $$\prod_{cyc} (a_i^3+a_{i+1}+1) \geq\prod_{cyc} (a_i^3+a_{i}+1)\geq \prod_{cyc} (a_ia_{i+1}^2+a_{i+1}+1)$$ che è la tesi.
11.3 Pit
Testo nascosto:
Siano $a=x+y$, $b=x+z$ e $c=y+z$ con $x,y$ e $z$ reali positivi. Usando la formula di Erone otteniamo $$2(x+y+z)=P\leq 2A^2=\frac{2\cdot (x+y+z)\cdot 2x\cdot 2y\cdot 2z}{8}$$ $$\Rightarrow xyz\geq 1$$
Mentre la tesi diventa $$(2x+y+z)(x+2y+z)(x+y+2z)\leq(x^2+y^2+2xy)(x^2+z^2+2xz)(y^2+z^2+2yz)$$
Usando $xyz\geq 1$, otteniamo $$(2x+y+z)(x+2y+z)(x+y+2z)\leq xyz(2x+y+z)(x+2y+z)(x+y+2z)=$$ $$=(2x^2+xy+xz)(xy+2y^2+yz)(xz+yz+2z^2)$$
quindi è sufficiente mostrare $$(2x^2+xy+xz)(xy+2y^2+yz)(xz+yz+2z^2)\leq (x^2+y^2+2xy)(x^2+z^2+2xz)(y^2+z^2+2yz)$$ $$\iff \frac{(2x^2+xy+xz)(xy+2y^2+yz)(xz+yz+2z^2)}{x^2y^2z^2}\leq \frac{(x^2+y^2+2xy)(x^2+z^2+2xz)(y^2+z^2+2yz)}{x^2y^2z^2}$$ $$\iff \frac{2x^2+xy+xz}{x^2}\frac{xy+2y^2+yz}{y^2}\frac{xz+yz+2z^2}{z^2}\leq \frac{x^2+y^2+2xy}{xy}\frac{x^2+z^2+2xz}{xz}\frac{y^2+z^2+2yz}{yz}$$ $$\iff \left(\frac{z}{x} +2+\frac{y}{x}\right)\left(\frac{x}{y}+ 2+\frac{z}{y}\right)\left(\frac{y}{z}+ 2+\frac{x}{z}\right)\leq \left(\frac{x}{y} +2+\frac{y}{x}\right)\left(\frac{z}{x}+ 2+\frac{x}{z}\right)\left(\frac{y}{z}+ 2+\frac{z}{y}\right)$$
Siano $l=\frac{x}{y}$, $m=\frac{y}{z}$ e $n=\frac{z}{x}$, l'ultima disuguaglianza possiamo scriverla come $$\left(l+2+\frac{1}{m}\right)\left(m+2+\frac{1}{n}\right)\left(n+2+\frac{1}{l}\right) \leq \left(l+2+\frac{1}{l}\right)\left(m+2+\frac{1}{m}\right)\left(n+2+\frac{1}{n}\right)$$ ma quest'ultima vale per il secondo lemma dato che $f(x)=2+\frac{1}{x}$ è decrescente, da cui la tesi.

12.1 Pit
Testo nascosto:
Per $a=0$ e $a=2$ possiamo prendere rispettivamente $f(x)=x$ e $f(x)=x+\frac{1}{x}$. Supponiamo ora che $a$ sia diverso da $0$ e da $2$.
Sia $f(x)=\frac{P(x)}{Q(x)}$ con $(P(x),Q(x))=1$ e $Q(x)$ monico e di grado $n$, riscriviamo la condizione del testo come $$(P(x)^2-aQ(x)^2)Q(x^2)=P(x^2)Q(x)^2$$ $$\Rightarrow Q(x^2)|P(x^2)Q(x)^2$$ ma essendo $P$ e $Q$ coprimi, si ha $$Q(x^2)|Q(x)^2$$ Dato che questi ultimi due sono hanno lo stesso grado e coefficiente direttore, si ha $$Q(x^2)=Q(x)^2$$ Supponiamo per assurdo che $Q(x)$ non sia un monomio e sia $a_lx^l$ il secondo termine di grado più alto non nullo. Allora nell'espansione di $Q(x)^2$ compare il termine $2a_lx^{n+l}$ che non invece non compare in $Q(x^2)$, assurdo $$\Rightarrow Q(x)=x^n$$
Sia $m$ il grado di $P(x)$
Distinguiamo ora due casi:
$1)\; n=0$ $$P(x)^2=P(x^2)+a$$ e come prima se $P$ non fosse un monomio, avremmo (detto $a_lx^l$ il secondo termine di grado più alto non nullo) che nel $LHS$ compare un termine di grado $m+l$ che non compare a destra $\Rightarrow P(x)=\alpha x^m$ con $m\geq 1$ essendo $f$ non costante $$\Rightarrow \alpha^2x^{2m}=\alpha x^{2m}+a\Rightarrow a=0$$ assurdo.
$2)\; n\geq 1$ $$P(x)^2=P(x^2)+ax^{2n}$$ dato che $(P(x),x^n)=1$, si ha $P(0)\neq 0$. Ponendo nella condizione $x\mapsto 0$ si ha $P(0)^2=P(0)\Rightarrow P(0)=1$.
Supponiamo che $P(x)$ abbia almeno $3$ coefficienti non nulli. Siano $a_lx^l$ e $a_kx^k$ i secondi termini di grado rispettivamente più alto e più basso (eventualmente $l=k$ se $P$ ha esattamente $3$ coefficienti).
Allora nel $LHS$ il secondo termine di grado più alto è $m+l$. Questo non compare in $P(x^2)$ e quindi $2n=m+l$.
Similmente il secondo termine di grado più basso è $k$. Questo non compare in $P(x^2)$ e quindi $2n=k$.
Quindi avremmo $$2n=k\leq l<l+m=2n$$ assurdo.
Se invece $P$ ha esattamente due coefficienti non nulli, si ha $$ax^{2n}+\alpha x^{2m}+1=\left(\alpha x^m+1\right)^2=\alpha^2 x^{2m}+1+2\alpha x^m$$ e per confronto si ottiene $a=2$, assurdo.
Rimane il caso in cui $P$ è un monomio, da $P(0)\neq 0$ si ottiene $P$ costante che porta ad $n=0$, assurdo anche questo caso.
I due unici valori di $a$ sono quindi $a=0$ e $a=2$.
12.2 Pit
Testo nascosto:
Se uno dei quattro polinomi è nullo otteniamo subito le soluzioni $$(P,Q,R,S)=\left(a,\frac{1}{a},0,S(x)\right);\; \left(a,\frac{1}{a},R(x),0\right);\; \left(0,Q(x),-a,\frac{1}{a}\right);\; \left(P(x),0,-a,\frac{1}{a}\right)$$ con $a$ reale non nullo.
Supponiamo ora che nessuno dei 4 polinomi sia nullo.
Sia $T(x,y,z,t)$ con $xy-zt=1$ l'equazione del testo.
Confrontando $T(x,y,xy-1,1)$ e $T(y,x,xy-1,1)$ si ottiene $$P(x)Q(y)=P(y)Q(x)$$ $\Rightarrow \frac{P(x)}{Q(x)}$ non dipende da $x\Rightarrow Q(x)=cP(x)$. Allo stesso modo otteniamo $S(x)=dR(x)$.
Sia $n$ il grado di $P(x)$ e $A(x)=P(x)P(x^n)$, per ogni $x$ questo dipende solo da $x^{n+1}\Rightarrow A(x)$ è un polinomio in $x^{n+1}$.
Supponiamo per assurdo che $P$ non sia un monomio e siano $\alpha x^n$ e $\beta x^l$ i termini di grado massimo. Allora il secondo termine di grado più alto in $P(x)P(x^n)$ è $\alpha\beta x^{n^2+l}$ $$\Rightarrow n+1|n^2+l\Rightarrow n+1|n(n+1)+l-n\Rightarrow n+1|n-l$$ ma $0<n-l\leq n$ assurdo.
Quindi $P(x)$ è un monomio e allo stesso modo lo è $R(x)$ $$\Rightarrow a(xy)^n-b(zt)^m=1\Rightarrow a(x+1)^n-bx^m=1$$ per ogni $x$ (dato che vale per infiniti valori) se uno tra $n$ e $m$ è nullo, lo è anche l'altro e otteniamo le soluzioni $(P,Q,R,S)=\left(a,b,c,\frac{ab-1}{c}\right)$ per ogni terna di reali non nulli $a,b,c$.
Supponiamo infine che sia $n$ che $m$ siano positivi. Dovendosi semplificare il termine di grado massimo in $a(x+1)^n-bx^m=1$, deve essere uguale sia il grado che il coefficiente, quindi $n=m$ e $a=b$. Ponendo $x\mapsto 0$ si ha $a=1\Rightarrow (x+1)^n-x^n=1$.
Se $n\geq 2$, dallo sviluppo di $(x+1)^n$ si ottiene un termine $nx^{n-1}$ che non viene semplificato, assurdo $\Rightarrow n=1$. L'ultima soluzione quindi è $(P,Q,R,S)=(x,x,x,x)$.
12.3 Pit
Testo nascosto:
Sia $n$ il grado di $P$, confrontando i gradi dell'equazione del testo di ottiene $\deg \; Q=n$. Se $n=0$ allora la tesi è banalmente vera, supponiamo quindi $n\geq 1$.
Supponiamo per assurdo che $R(x)=P(x)-Q(x)$ sia non nullo e abbia grado $k$ tale che $0<k\leq n-1$, allora otteniamo $$Q(P(x))-Q(Q(x))+R(P(x))=P(P(x))-Q(Q(x))=0$$ $$\Rightarrow Q(P(x))-Q(Q(x))=-R(P(x))$$ $$\Rightarrow \deg \left(Q(P(x))-Q(Q(x))\right)=\deg\left(R(P(x))\right)=kn$$
Sia $Q(x)=x^n+a_{n-1}x^{n-1}+...+a_1x+a_0$, $$\Rightarrow Q(P(x))-Q(Q(x))=\left[P(x)^n-Q(x)^n\right]+...+a_1\left[P(x)-Q(x)\right]$$
ma in quest'ultima espressione il grado di tutte i termini tra parentesi quadre (tranne la prima) è minore o uguale a $n^2-n$, mentre nella prima si ha $$P(x)^n-Q(x)^n=R(x)\left[P(x)^{n-1}+P(x)^{n-2}Q(x)+...+Q(x)^{n-1}\right]$$ che ha grado $n^2-n+k$, infatti il termine di grado massimo nella parentesi non si semplifica ed è uguale a $nx^{n^2-n}$ $$kn=n^2-n+k>n^2-n\Rightarrow k>n-1$$ assurdo.
Rimane da trattare il caso $k=0\Rightarrow P(x)=Q(x)+c$ con $c\neq 0$. Sostituiamo questo nel testo e otteniamo $$Q(Q(x)+c)+c=Q(Q(x))$$ ma essendo $Q$ non costante, questo assume infiniti valori $$\Rightarrow Q(x+c)+c=Q(x)$$ $$\Rightarrow (x+c)^n+a_{n-1}(x+c)^{n-1}+...+a_1(x+c)+a_0+c=x^n+a_{n-1}x^{n-1}+...+a_1x+a_0$$ per $n\geq 2$, confrontando i coefficienti di grado $n-1$ si ottiene $$nc+a_{n-1}=a_{n-1}\Rightarrow nc=0$$ assurdo. Infine se $n=1$ si ha $$x+c+a_0+c=x+a_0\Rightarrow c=0$$ da cui la tesi.

13.1 Pit
Testo nascosto:
Dimostriamo la tesi per induzione su $n\geq 2$.
Passo base $n=2$.
Bisogna dimostrare che $$a_1a_2\leq\frac{1}{4}$$
con $a_1+a_2=1$, ma questa è chiaramente vera per $AM-GM$.
Passo base $n=3$.
Bisogna mostrare che $a_2(a_1+a_3)\leq \frac{1}{4}$
con $a_2+a_1+a_3=1$, ma anche questa è vera per $AM-GM$
Supponiamo ora che la tesi sia vera per $n$ e mostriamola per $n+1$.
Usando l'ipotesi induttiva sulla $n$-upla $(a_2,a_1+a_3,a_4,...,a_n,a_{n+1})$ (che rispetta le ipotesi) si ottiene $$\frac{1}{4}\geq a_2(a_1+a_3)+(a_1+a_3)a_4+a_4a_5+...+a_na_{n+1}=$$ $$=a_1a_2+a_2a_3+a_3a_4+...+a_na_{n+1}+a_1a_4\geq a_1a_2+a_2a_3+a_3a_4+...+a_na_{n+1}$$
che è la tesi.
13.2 Pit
Testo nascosto:
Siano $a=\frac{2x}{y},b=\frac{2y}{z}$ e $c=\frac{2z}{x}$ con $x,y,z>0.$ Allora
$$\sum_{cyc} \frac{a^2}{\sqrt{(1+a^3)(1+b^3)}} \geq \frac{4}{3}\iff\sum_{cyc} \frac{x^2z^2}{\sqrt{yz(y^3+8x^3)(z^3+8y^3)}} \geq \frac{1}{3}\iff$$
$$\sum_{cyc} \frac{x^2z^2}{\sqrt{yz(y+2x)(y^2+4x^2-2xy)(z+2y)(z^2+4y^2-2yz)}} \geq \frac{1}{3}\iff$$
$$\sum_{cyc} \frac{x^2z^2}{\sqrt{(y^2+2xy)(y^2+4x^2-2xy)(z^2+2yz)(z^2+4y^2-2yz)}} \geq \frac{1}{3}$$

Per $AM≥GM$ applicato a $(y^2+2xy,y^2+4x^2−2xy),$ si ha $\sqrt{(y^2+2xy)(y^2+4x^2-2xy)}\leq y^2+2x^2$, allo stesso modo si ottengono tutte le versioni cicilche. Resta quindi da dimostrare
$$\frac{x^2z^2}{(y^2+2x^2)(z^2+2y^2)}+\frac{x^2y^2}{(z^2+2y^2)(x^2+2z^2)}+\frac{y^2z^2}{(x^2+2z^2)(y^2+2x^2)}\geq\frac{1}{3}$$
Moltiplichiamo entrambi i membri per $3(x^2+2z^2)(y^2+2x^2)(z^2+2y^2)$, quindi
$$\frac{x^2z^2}{(y^2+2x^2)(z^2+2y^2)}+\frac{x^2y^2}{(z^2+2y^2)(x^2+2z^2)}+\frac{y^2z^2}{(x^2+2z^2)(y^2+2x^2)}\geq\frac{1}{3}\iff$$
$$6(x^4y^2+y^4z^2+x^2z^4)+3(x^4z^2+x^2y^4+y^2z^4)\geq 4(x^4y^2+y^4z^2+x^2z^4)+2(x^4z^2+x^2y^4+y^2z^4)+9x^2y^2z^2$$
$$\iff 2(x^4y^2+y^4z^2+x^2z^4)+(x^4z^2+x^2y^4+y^2z^4)\geq 9x^2y^2z^2$$

Per $AM≥GM$ applicata prima su $(x^4y^2,y^4z^2,x^2z^4)$ e poi su $(x^4z^2,x^2y^4,y^2z^4)$ si ha
$$(x^4y^2+y^4z^2+x^2z^4)\geq 3x^2y^2z^2\Rightarrow 2(x^4y^2+y^4z^2+x^2z^4)\geq 6x^2y^2z^2$$
$$(x^4z^2+x^2y^4+y^2z^4)\geq 3x^2y^2z^2$$
sommando queste due, si ottiene la tesi.
13.3 Pit
Testo nascosto:
Siano $a=\left(\frac{x_1}{2017}\right)^{\frac{1}{4}},b=\left(\frac{x_2}{2017}\right)^{\frac{1}{4}},c=\left(\frac{x_3}{2017}\right)^{\frac{1}{4}},d=\left(\frac{x_4}{2017}\right)^{\frac{1}{4}}$. La condizione diventa
$$\frac{1}{x_1+2017}+\frac{1}{x_2+2017}+\frac{1}{x_3+2017}+\frac{1}{x_4+2017}=\frac{1}{2017}$$
e la tesi
$$\frac{\sqrt[4]{x_1x_2x_3x_4}}{4-1}\geq 2017$$
che corrisponde alla disuguaglianza 9.3. per $n=4$.

14.1 Pit
Testo nascosto:
Dalla seconda equazione otteniamo $f(1^3)=f(1)^3⇒f(1)=1$ e dalla prima otteniamo induttivamente $f(m)=m$ e $f(x+m)=f(x)+m$ per ogni $x$ razionale positivo e $m$ intero positivo.
Ora fissato $x$ prendiamo un intero positivo $n$ tale che $nx^2$ sia un numero intero (in particolare se $x=\frac{a}{b}$ con $a,b$ interi positivi, possiamo prendere $n=b^2$), dalle due equazioni otteniamo
$$f((x+n)^3)=f(x+n)^3=(f(x)+n)^3=f(x)^3+3nf(x)^2+3n^2f(x)+n^3$$

e dato che $3n^2x+3nx^2+n^3$ è un intero positivo per come abbiamo scelto $n$, si ha
$$f((x+n)^3)=f(x^3+3n^2x+3nx^2+n^3)=f(x^3)+3n^2x+3nx^2+n^3=f(x)^3+3n^2x+3nx^2+n^3$$
da cui
$$f(x)^3+3nf(x)^2+3n^2f(x)+n^3=f(x)^3+3n^2x+3nx^2+n^3\Rightarrow f(x)^2+nf(x)-(x^2+nx)=0$$
$$⇒(f(x)-x)(f(x)+x+n)=0\Rightarrow f(x)=x$$

per ogni $x$ razionale positivo. Questa funzione verifica entrambe le equazioni e quindi è l'unica soluzione.
14.2 Davide Di Vora
Testo nascosto:
Le soluzioni sono $f(x)=x$, $f(x)=0$ e $f(x)=2$, che sostituendo verificano.

Sia $P(x;y)$ l'equazione funzionale del testo.
Da $P(0;0)$ ottengo $f(0)=0$ o $f(0)=2$. Suddivido il problema in due casi:

1. $f(0)=2$
Da $P(x;0)$ ottengo la soluzione $f(x)=2$.

2. $f(0)=0$
Da $P(2;2)$ ottengo $f(2)=2$ o $f(2)=0$, mentre da $P(1;1)$ ottengo $f(2)=−f(1)^2+3f(1)$. Ci sono dunque i seguenti casi:

a. $f(1)=0$ e $f(2)=0$
Da $P(x;1)$ ottengo
$$f(x+1)=2f(x)$$

Da $P(1;−1)$ ottengo $f(−1)=0$
Da $P(x+1;y)$, utilizzando anche il testo, ottengo
$$2f(xy)+f(y)=f(xy+y)$$

ma ponendo nell'ultima relazione $x\mapsto\frac{1}{x}$ con $x\neq 0$, ho $$f(x)=f(x+1)=2f(x)$$ e ottengo soluzione $f(x)=0$.

b. $f(1)=3$ e $f(2)=0$
Da $P(x,1)$ ottengo $$f(x+1)+f(x)=3$$ $$\Rightarrow f(x+1)+f(x)+3=f(x+1)+f(x+2)\Rightarrow f(x)=f(x+2)$$
Da $P\left(2,\frac{1}{2}\right)$ ottengo $$f\left(\frac{1}{2}\right)=f\left(2+\frac{1}{2}\right)=3+f\left(\frac{1}{2}\right)$$
che è assurdo.

c. $f(1)=1$ e $f(2)=2$
Da $P(x;1)$ ottengo
$$f(x+1)=f(x)+1$$

Da $P(x+1;y)$ ottengo
$$f(x+y)+f(x)f(y)=f(xy+y)+f(x)$$

Sfruttando ora il testo ottengo
$$f(xy)+f(y)=f(xy+y)$$

Siano ora $a$ e $b$ due reali qualsiasi con a$≠0.$
Pongo ora $y⟵a$ e $x⟵\frac{b}{a}$ e ottengo
$$f(a+b)=f(a)+f(b)$$

ma $f(0)=0$ e quindi
$$f(x+y)=f(x)+f(y)$$

$∀x,y$ reali.
Da $P(x;x)$ ottengo ora $f(x^2)=f(x)^2≥0$ e quindi $f$ non è densa nel piano, da cui segue che l'unica soluzione dell'equazione di Cauchy è $f(x)=kx$, ma essendo $f(1)=1$ ottengo che l'unica soluzione in questo caso è $f(x)=x$.

d. $f(1)=2$ e $f(2)=2$
Da $P(x;1)$ ottengo $f(x)=2$, che è assurdo poichè $f(0)=0$ e quindi in questo caso non ci sono soluzioni.
14.3 Pit
Testo nascosto:
Sia $S$ l'insieme di numeri reali $t$ tali che che $f(x)≥tx$ per ogni funzione $f∈F$.
Mostriamo prima che $S$ non è vuoto, usando $\frac{x}{3}$ al posto di $x$ nella disuguaglianza del testo, otteniamo
$$f(x)\geq f\left(f\left( \frac{2x}{3}\right)\right)+\frac{x}{3}\geq\frac{x}{3}\Rightarrow \frac{1}{3}\in S$$

Mostriamo ora che gli elementi in $S$ sono limitati superiormente, consideriamo la funzione $f(x)=\frac{x}{2}$, questa rispetta la disuguaglianza del testo dato che
$$f(3x)=\frac{3x}{2}=f(f(2x))+x$$

otteniamo quindi che tutti gli elementi $t$ in $S$ sono tali che $\frac{x}{2}≥tx⇒t≤\frac{1}{2}$
Sia $b=\sup S$, supponiamo che $b∉S$, esiste quindi almeno una funzione $f∈F$ e un reale positivo $x_0$ tali che $\frac{f(x_0)}{x_0}<b.$
Sia $k=\frac{f(x_0)}{x_0}$ e $\{c_n\}_{n≥0}$ una successione di reali positivi appartenenti a $S$ che converge a $b$. Si ha
$$c_n≤k<b$$per ogni $n$ intero non negativo. Ma dato che $\lim\limits_{n\to\infty} c_n=b$, per il teorema del confronto si ha $k=\lim\limits_{n\to\infty} k=b$ che è assurdo dato che $k<b$, quindi deve valere $b∈S⇒b$ è il massimo di $S$ e vale inoltre $b≤\frac{1}{2}$.
Sfruttando la disuguaglianza del testo con $\frac{x}{3}$ al posto di $x$ e la disuguaglianza $f(x)≥bx$, si ottiene
$$f(x)\geq f\left(f\left( \frac{2x}{3}\right)\right)+\frac{x}{3}\geq bf\left(\frac{2x}{3}\right)+\frac{x}{3}\geq \frac{2b^2x}{3}+\frac{x}{3}=x\frac{2b^2+1}{3}\Rightarrow \frac{2b^2+1}{3}\in S$$

essendo $b$ il massimo, si ha $b\geq \frac{2b^2+1}{3}\Rightarrow 2b^2-3b+1\leq 0\Rightarrow \frac{1}{2}\leq b\leq 1$ che unito a $b≤\frac{1}{2}$ dà $b=\frac{1}{2}$ che è il valore cercato.

15.1 TheRoS
Testo nascosto:
Sia $k=\frac{a}{b}$ che quindi per ipotesi è $≥1$. Considerando $a=bk$ ottengo che:
$$\begin{align}
b(\sqrt{k^2 +1}+\sqrt[3]{k^3+1}+\sqrt[4]{k^4+1})\le (3k+1)b\iff \\
\iff
\sqrt{k^2 +1}+\sqrt[3]{k^3+1}+\sqrt[4]{k^4+1}\le 3k+1
\end{align}$$

A questo punto si definisca $s_l(x)=\sqrt[l]{x^l+1}-x$ con $l∈N$. Perché la tesi sia vera vogliamo $s_2(k)+s_3(k)+s_4(k)≤1$. Notiamo che le funzioni $s_l(x)$ sono decrescenti in $\mathbb{R}+$, da cui
$$s_2(k)+s_3(k)+s_4(k)≤s_2(1)+s_3(1)+s_4(1)$$
Rimane quindi da dimostrare che $\sqrt[2]{2}+\sqrt[3]{2}+\sqrt[4]{2}\leq 4$, ma questa è vera perché $$\sqrt[2]{2}+\sqrt[3]{2}+\sqrt[4]{2}<\sqrt[2]{2.25}+\sqrt[3]{2.197}+\sqrt[4]{2.0736}=\sqrt[2]{(1.5)^2}+\sqrt[3]{(1.3)^3}+\sqrt[4]{(1.2)^4}=1.5+1.3+1.2=4$$
15.2 Pit
Testo nascosto:
Fissiamo $k\geq 1$ intero e consideriamo $$f(x)=\sqrt[k]{x^k+1}-1+x\left(1-\sqrt[k]{2}\right)$$ con $0\leq x\leq 1$. Allora si ha $$f''(x)=(k-1)x^{k-2}(x^k+1)^{\frac{1}{k}-2}\geq 0$$ $\Rightarrow f(x)$ è convessa e quindi assume il suo massimo in uno degli estremi del suo dominio $$\Rightarrow f(x)\leq \max(f(0),f(1))\Rightarrow f(x)\leq 0$$ $$\Rightarrow \sqrt[k]{x^k+1}\leq 1+x\left(\sqrt[k]{2}-1\right)$$
Prendendo ora $l$ e $m$ tali che $l \geq m \geq 0$, si ha (prendendo $x=\frac{m}{l}$ nell'ultima disuguaglianza) $$\sqrt[k]{l^k+m^k} \leq l + m(\sqrt[k]{2} -1)$$
$WLOG\; a\geq b\geq c$
Usando tre volte questa disuguaglianza nel $LHS$ del testo, otteniamo $$\sqrt[n]{a^n+b^n} + \sqrt[n]{b^n+c^n} + \sqrt[n]{c^n+a^n}\leq (2a+b)-(b+2c)+\sqrt[n]{2}\cdot (b+2c)=$$ $$2a-2c+\sqrt[n]{2}\cdot (b+2c)-1-\frac{\sqrt[n]{2}}{2}+1+\frac{\sqrt[n]{2}}{2}=(2a-2c-1)-\frac{\sqrt[n]{2}}{2}\cdot (1-2b-4c)+1+\frac{\sqrt[n]{2}}{2}$$ sostituendo $b=1-a-c$ $$(2a-2c-1)-\frac{\sqrt[n]{2}}{2}\cdot (2a-2c-1)+1+\frac{\sqrt[n]{2}}{2}=$$ $$=(2a-2c-1)\left(1-\frac{\sqrt[n]{2}}{2}\right)+1+\frac{\sqrt[n]{2}}{2}$$
Essendo $n\geq 2$, si ha $$\sqrt[n]{2}<2\Rightarrow 1-\frac{\sqrt[n]{2}}{2}>0$$ Per la disuguaglianza triangolare vale $$1=a+b+c>2a\Rightarrow 2a-1-2c<0$$ e da queste si ottiene $$\sqrt[n]{a^n+b^n} + \sqrt[n]{b^n+c^n} + \sqrt[n]{c^n+a^n}\leq (2a-2c-1)\left(1-\frac{\sqrt[n]{2}}{2}\right)+1+\frac{\sqrt[n]{2}}{2}<1+\frac{\sqrt[n]{2}}{2}$$ che è la tesi.
15.3 Pit
Testo nascosto:
$WLOG\; c=\min(a,b,c)$
Siano $x=a+\frac{c}{2}$ e $y=b+\frac{c}{2}$. Allora si ha $$x^2=a^2+\frac{c^2}{4}+ac\geq a^2+c^2$$ $$y^2=b^2+\frac{c^2}{4}+bc\geq b^2+c^2$$ e $$x^2+y^2=a^2+b^2+\frac{c^2}{2}+ac+bc\geq a^2+b^2$$ da cui $$\sum_{cyc} \frac{1}{a^2+b^2} - \frac{10}{(a+b+c)^2}\geq \frac{1}{x^2}+\frac{1}{y^2}+\frac{1}{x^2+y^2}-\frac{10}{(x+y)^2}=$$ $$=\frac{(x-y)^2(x^4+y^4+4x^3y+4xy^3+x^2y^2)}{x^2y^2(x^2+y^2)(x+y)^2}\geq 0$$ da cui la tesi.

16.1 Pit
Testo nascosto:
Sia $a=P(1)$, ponendo $x\mapsto 1$ si ottiene $2a=a^2-2$.
Supponiamo prima che $P(x)$ non sia costante.
Sia $n$ la molteplicità con la quale $x-1$ divide $P(x)-a$, abbiamo chiaramente $n\geq 1$. Scriviamo $P(x)$ come $(x-1)^nQ(x)+a$ con $c=Q(1)\neq 0$ e sostituiamolo nell'equazione. $$2(2x^2-2)^nQ(2x^2-1)+2a=(x-1)^{2n}Q(x)^2+2a(x-1)^nQ(x)+a^2-2$$$$\Rightarrow 2^{n+1}(x+1)^n(x-1)^nQ(2x^2-1)=(x-1)^{2n}Q(x)^2+2a(x-1)^nQ(x)$$$$\Rightarrow 2^{n+1}(x+1)^nQ(2x^2-1)=(x-1)^nQ(x)^2+2aQ(x)$$ ponendo in quest'ultima equazione $x\mapsto 1$ si ottiene $$2^{2n+1}c=2ac\Rightarrow a=2^{2n}$$ ma questo è assurdo visto che da $a^2-2a-2=0$ si ottiene $a=1+\sqrt{3}$ o $a=1-\sqrt{3}\Rightarrow P(x)$ è costante.
Quindi le uniche due soluzioni sono $P(x)=1+\sqrt{3}$ e $P(x)=1-\sqrt{3}$ che chiaramente soddisfano.
16.2 Pit
Testo nascosto:
Sia $f(x)$ un polinomio che soddisfa la condizione del testo, allora avremo $$f(x)^2=f(x^2+1)-1=f((-x)^2+1)-1=f(-x)^2\Rightarrow (f(x)-f(-x))(f(x)+f(-x))=0$$ almeno una delle due parentesi si annulla per infiniti valori di $x$, e quindi per ogni valore.
$\cdot$ Se questa è la seconda, otteniamo $f(0)+f(-0)=0\Rightarrow f(0)=0$. Consideriamo la successione $a_{n+1}=a_n^2+1$ e $a_0=0$.
Mostriamo che $f(a_i)=a_i$ per induzione su $i\geq 0$. Il passo base lo abbiamo già mostrato, passiamo quindi a quello induttivo.
Ponendo $x\mapsto a_n$ si ha $$f(a_{n+1})=f(a_n^2+1)=f(a_n)^2+1=a_n^2+1=a_{n+1}$$
da cui la tesi.
Gli $a_i$ sono tutti distinti e quindi $f(x)$ coincide con $x$ per infiniti valori $\Rightarrow f(x)=x$.
$\cdot$ Se invece la prima parentesi è quella nulla per ogni $x$ si ha che $f$ è pari $\Rightarrow f(x)=g(x^2+1)$ per un qualche polinomio $g$.
Sostituendo nell'equazione e chiamando $y=x^2+1$ otteniamo $$g(y^2+1)=g(y)^2+1$$ ma $y$ assume infiniti valori $$\Rightarrow g(x^2+1)=g(x)^2+1$$ per ogni $x$.
Sia ora $T_i$ una successione di polinomi tali che $T_0(x)=x$ e $T_{n+1}=T_n(x^2+1)$ e mostriamo che i polinomi di questa successione sono tutte e sole le soluzioni al problema. Chiaramente tutti questi sono soluzioni. Mostriamo che sono le uniche.
Mostriamo in particolare per induzione su $k\geq 0$ che se $P$ è soluzione e ha grado $\leq 2^k$, allora è di quella forma.
Passo base $k=0$.
Se $P(x)=c$ è costante, otteniamo $$c=c^2+1$$ che non ha soluzioni. Se invece $P$ è di primo grado, non può essere pari e quindi per quanto dimostrato prima deve essere $P(x)=x=T_0(x)$.
Passo induttivo. Supponiamo che la tesi valga per $k$ e mostriamo che vale per $k+1$.
Se $P$ è soluzione e $\deg P(x)\leq 2^k$ allora la tesi è vera per ipotesi induttiva. Supponiamo quindi che $2^k<\deg P(x)\leq 2^{k+1}$.
Essendo $P$ non lineare, per quanto mostrato prima esiste $P_0(x)$ che rispetta la condizione del testo e tale che $P(x)=P_0(x^2+1)$. Confrontando i gradi di questa espressione otteniamo $\deg P_0(x)=\frac{\deg P(x)}{2}\leq 2^k$.
Quindi per ipotesi induttiva $P_0(x)=T_i(x)$ per qualche $i\Rightarrow P(x)=P_0(x^2+1)=T_i(x^2+1)=T_{i+1}(x)$ che completa l'induzione.
16.3 Pit
Testo nascosto:
Sia $P(x)=a_nx^n+a_{n-1}x^{n-1}...+a_1x+a_0$ con gli $a_i$ razionali, sia $k$ il minimo comune multiplo dei denominatori dei coefficienti di $P$.
Quindi avremo $P(x)=\frac{b_nx^n+b_{n-1}x^{n-1}...+b_1x+b_0}{k}$ con i $b_i$ interi. Sia infine $p$ un primo che non divide né $k$ né $b_n$ e poniamo $y=\frac{1}{p}$.
Sappiamo quindi che esiste un razionale $\frac{c}{d}$ con $MCD(c,d)=1$ tale che $P\left(\frac{c}{d}\right)=\frac{1}{p}$, quindi
$$\left(b_n\frac{c^n}{d^n}+...+b_1\frac{c}{d}+b_0\right)\cdot p=k$$$$\Rightarrow (b_nc^n+b_{n-1}c^{n-1}d+...+b_1cd^{n-1}+b_0d^n)\cdot p=kd^n$$ $$\Rightarrow p|kd$$ per come abbiamo scelto $p$ abbiamo quindi $p|d$. Essendo $c$ e $d$ coprimi, $p$ non divide $c$.
Supponiamo per assurdo $n\geq 2 \Rightarrow p^2|kd^n$$$\Rightarrow p^2|(b_nc^n+b_{n-1}c^{n-1}d+...+b_1cd^{n-1}+b_0d^n)\cdot p$$ $$p|b_nc^n+b_{n-1}c^{n-1}d+...+b_1cd^{n-1}+b_0d^n$$ ma dato che $p| d$, otteniamo $$p|b_nc^n$$ che è assurdo visto che $p$ non divide né $b_n$ né $c\Rightarrow n=1$ e quindi $P$ è un polinomio affine.

17.1 Pit
Testo nascosto:
Mostriamo che
$$\frac{1+x^{2k}}{1+x^{4k}}<\frac{1}{x^k}$$

Facendo denominatore comune diventa
$$x^k+x^{3k}<1+x^{4k}\Leftrightarrow (1-x^{3k})(1-x)>0$$
che è vera dato che $0<x<1$.
Usando questa disuguaglianza anche sui termini con la $y$ otteniamo
$$\left( \sum_{k=1}^n \frac{1+x^{2k}}{1+x^{4k}} \right) \left( \sum_{k=1}^n \frac{1+y^{2k}}{1+y^{4k}} \right) < \left( \sum_{k=1}^n \frac{1}{x^k} \right) \left( \sum_{k=1}^n \frac{1}{y^k} \right) =$$$$=\frac{1-x^n}{x^n(1-x)}\frac{1-y^n}{y^n(1-y)}=\frac{y^n}{x^n(1-x)}\frac{x^n}{y^n(1-y)}=\frac{1}{(1-x)(1-y)}$$

che è la tesi.
17.2 Pit
Testo nascosto:
Sia $f(1)=c$ e supponiamo $c>1$. Con $a↦1$ sappiamo che esiste un triangolo con lati $1,f(b),f(b+(c−1))$, per la disuguaglianza triangolare questo implica $f(b)=f(b+(c−1))$ che induttivamente porta a
$$f(b)=f(b+k(c−1))$$
per ogni $k$ naturale. Quindi l'immagine di $f$ ha al massimo $c−1$ elementi $⇒f$ è limitata ma
$$a<f(b)+f(b+f(a)−1)$$
per ogni $a$ e $b$. Questo è assurdo essendo il membro di destra limitato mentre quello di destra no $⇒f(1)=c=1$.
Con $b↦1$ otteniamo $f(f(a))=a$ quindi $f$ è bigettiva.
Sia $f(2)=x≥2$. Da $a↦2$ sappiamo che esiste un triangolo di lati $2,f(b),f(b+x−1)$, quindi $h(b)=f(b+x−1)−f(b)$ vale $0,1$ o $−1$ per ogni $b$. Non può essere $0$ altrimenti per iniettività si avrebbe $x=1$ che è assurdo. Supponiamo che esista un intero $y$ tale che $h(y)=1$ e $h(y+x−1)=−1$ (o viceversa). In ogni caso abbiamo
$$h(y)+h(y+x−1)=0⇒f(y+x−1)−f(y)+f(y+2(x−1))−f(y+x−1)=0$$
$$⇒f(y+2(x−1))=f(y)⇒y+2(x−1)=y⇒x=1$$
assurdo $⇒h(b)=h(b+k(x−1))$.
Se per qualche $y$ intero positivo $h(y)=−1,$ avremmo $f(y+k(x−1))=f(y)−k$, prendendo $k$ arbitrariamente grande il membro di destra diventerebbe negativo mentre quello di destra sarebbe positivo $⇒h(b)=1$ per ogni $b$ intero positivo.
$$⇒f(b+k(x−1))=f(b)+k$$

con $b↦1$ otteniamo $f(1+k(x−1))=1+k.$ Al variare di $k$ il membro di destra assume tutti i valori interi positivi e dato che $f$ è iniettiva anche l'argomento del membro di sinistra deve farlo, questo è possibile solo se $x−1=1$ (dato che l'argomento è congruo a $1$ modulo $x−1$)$⇒x=2$ e $f(b)=b$.
Rimane da verificare che questa funzione soddisfa la condizione, ma questo è chiaro dato che per ogni $a$ e $b$ positivi si ha $$|b-a|<a+b-1<a+b$$
17.3 Pit
Testo nascosto:
Sia $P(x,y)$ la funzionale del testo. Sottraendo $P(x,y)$ e $P(y,x)$ membro a membro, si ha
$$(2x+y)g(y)+f(x)=f(y)+(2y+x)g(x)$$
con $y↦0$ si ottiene
$$f(x)=xg(x)−2xg(0)+f(0)$$
che sostituita nell'equazione precedente dà
$$(2x+y)g(y)+xg(x)−2xg(0)+f(0)=yg(y)−2yg(0)+f(0)+(2y+x)g(x)$$
$$⇒xg(y)−xg(0)=−yg(y)+yg(x)⇒x(g(y)−g(0))=y(g(x)−g(0))$$
per $x,y≠0$ si ha
$$\frac{g(x)-g(0)}{x}=\frac{g(y)-g(0)}{y}$$
$⇒\frac{g(x)-g(0)}{x}$ è costante $⇒g(x)=cx+g(0)$ per ogni $x≠0$ ma questa è chiaramente vera anche per $x=0$
$$⇒g(x)=cx+d$$
$$⇒f(x)=x(cx+d)−2dx+f(0)=cx^2−dx+e$$

Sostituiamo queste due nella funzionale del testo e otteniamo
$$c^2(x+y)^2−cd(x+y)+ce+d=cx^2−dx+e+2cxy+2xd+cy^2+dy$$
questa deve essere un'uguaglianza polinomiale identicamente vera ma il coefficiente di $y^2$ a sinistra è $c^2$ mentre a destra è $c$. Se $c=0$ otteniamo $g$ costante che nel testo porta a $g=0$ che a sua volta porta a $f$ nulla. Se invece $c=1$ otteniamo $d(2x+2y−1)=0⇒d=0$ che porta a $f(x)=x^2+e$. Le due coppie $(f(x),g(x))$ sono quindi $(0,0)$ e $(x,x^2+e)$ che verificano entrambe la funzionale.

18.1 TheRoS
Testo nascosto:
Per ogni $i$ appartenente a $(2,…,n)$ compiamo il seguente ragionamento.
$$\begin{equation}
a_i+1=a_i+\frac{1}{i-1}+\dots+\frac{1}{i-1}
\end{equation}$$

Dove $\frac{1}{i−1}$ compare $i−1$ volte. Applichiamo ora $AM−GM$ su questi i termini ottendo che:
$$\begin{equation}
\frac{a_i+1}{i}\geq(\frac{a_i}{(i-1)^{i-1}})^{1/i}\iff (a_i+1)^i\geq\frac{i^i\cdot a_i}{(i-1)^{i-1}}
\end{equation}$$

Grazie a questa osservazione possiamo scrivere che:
$$\begin{equation}
(a_2+1)^2\cdot\dots\cdot(a_n+1)^n\geq \frac{4a_2}{1}\cdot\dots\cdot\frac{i^i\cdot a_i}{(i-1)^{i-1}}\cdot\dots\cdot\frac{n^n\cdot a_n}{(n-1)^{n-1}}=n^n
\end{equation}$$
dove si ha l'uguaglianza se e solo se $a_i=\frac{1}{i-1}$. Ma se così fosse si avrebbe $$a_2 a_3 \cdots a_n <1$$ e quindi la disuguaglianza è stretta.
18.2 Pit
Testo nascosto:
Sia $P(x,y)$ l'equazione del testo.
Se $f(a)=a$ per qualche reale positivo, da $P(x,a)$ si ha $f(a)=0$ assurdo.
Se $f(a)<a$ per qualche reale positivo, da $P(a-f(a),a)$ si ha $f(2a-f(a))=0$ assurdo $\Rightarrow f(x)>x$.
Sia $g: \mathbb{R}^+ \to \mathbb{R}^+$ tale che $g(x)=f(x)-x\Rightarrow f(x)=g(x)+x$ e sostituendo questa nell'equazione del testo otteniamo $$g(g(y)+x+y)=g(x+y)+y$$$$g(g(x)+z)=g(z)+x$$ per ogni $x$, $z$ reali positivi con $z>x$. Sia $S(x,z)$ quest'ultima equazione.
Se per assurdo $g(a)=g(b)$ per due reali positivi, fissato un $t$ maggiore sia di $a$ che di $b$ da $S(a,t)$ e $S(b,t)$ otteniamo $a=b\Rightarrow g(x)$ è iniettiva. Ora fissiamo $x$ e $y$ e prendiamo $z>x+y$ da $S(x,g(y)+z)$, $S(y,z)$ e $S(x+y,z)$ otteniamo $$g(g(x)+g(y)+z)=x+g(g(y)+z)=x+y+g(z)=g(g(x+y)+z)$$ e usando l'iniettività di $g$ si ha $$g(x+y)=g(x)+g(y)$$$\Rightarrow g$ è additiva e limitata inferiormente $\Rightarrow g(x)=ax$ con $a$ reale positivo $\Rightarrow f(x)=bx$, sostituendo questa nell'equazione iniziale otteniamo $$bx+b^2y=bx+2by\Rightarrow b^2=2b\Rightarrow b=2\Rightarrow f(x)=2x$$
18.3 Parmenide
Testo nascosto:
Dimostro che l'unica soluzione è $f(x)=\frac{1}{x}$, che sostituendo verifica.
Sia $P(x;y)$ l'equazione funzionale del testo

Dimostro per prima cosa che $f$ è iniettiva:
da $P(x;1)$ si ottiene $f(f(x)^2)=x^3f(x)$

da cui, se $f(x)=f(y)$, si ha $\displaystyle{x^3=\frac{f(f(x)^2)}{f(x)}=\frac{f(f(y)^2)}{f(y)}=y^3 \Rightarrow x=y }$, quindi $f$ è iniettiva.


Dimostro ora che $f$ è moltiplicativa:
da $P(xy;1)$, $P(x;y)$, $P(y;f(x)^2)$ si ottiene $\displaystyle{f\left(f\left(xy\right)^2\right)=y^3x^3f\left(xy\right)=y^3f\left(f(x)^2y\right)=f\left(f(x)^2f(y)^2\right)}$
da cui, per l'iniettività, $f(xy)^2=f(x)^2f(y)^2 \Rightarrow f(xy)=f(x)f(y)$, quindi $f$ è moltiplicativa


A questo punto chiaramente si ha che $f(1)=1$ e $f(x^n)=f(x)^n$

In questo modo posso riscrivere $P(x;y)$ come $f\left(f(x)\right)^2f(y)=x^3f(x)f(y) \Rightarrow f\left(f(x)\right)=\displaystyle{\sqrt{x^3f(x)}}$

Sia ora $g(x)=xf(x)$.
Si ha $$g(g(x))=g(xf(x))=xf(x)f(xf(x))=xf\left(x^2f(x)\right)=xf\left(x^2\right)f\left(f(x)\right)=\displaystyle{xf\left(x\right)\sqrt{x^3f(x)}}=\left[xf(x)\right]^{\frac{5}{2}}=\left[g(x)\right]^{\frac{5}{2}}$$

e per induzione $g$ composto se stesso $n+1$ volte è $\left[g(x)\right]^{\left(\frac{5}{2}\right)^n}$

A questo punto fissiamo $x$ nell'equazione $g \circ g\circ … \circ g=\left[g(x)\right]^{\left(\frac{5}{2}\right)^n}$ dove nel $LHS$ la composizione è ripetuta $n+1$ volte.

Dimostro ora che $g(x)=1$:
$LHS$ è sempre razionale per come sono definite $f,g$, quindi anche $\left[g(x)\right]^{\left(\frac{5}{2}\right)^n}$ deve essere sempre razionale.
Supponiamo per assurdo $g(x)≠1$ e sia la scomposizione in fattori primi di $g(x)=p_1^{\alpha_1}...p_k^{\alpha_k}$ dove i $p_i$ sono primi distinti e gli $α_i$ sono interi diversi da $0$.
Allora la fattorizzazione di $g\circ g\circ …\circ g=p_1^{\left(\frac{5}{2}\right)^n \alpha_1}...p_k^{\left(\frac{5}{2}\right)^n \alpha_k}$
Quindi tutti gli esponenti dovrebbero essere sempre interi, ma ad esempio il primo esponente non lo è se $2^n∤α_1$, contraddizione.

Allora $g(x)=1$, che porta a $f(x)=\frac{1}{x}$.

19.1. Pit, giorgia17 e sab01
Testo nascosto:
Le funzioni possibili sono $0,x−1$ e $1−x$ che chiaramente soddisfano, mostriamo che sono le uniche.
Sia $P(x,y)$ l'equazione del testo.
$f$ è soluzione $\iff −f$ lo è, WLOG $f(0)≤0.$
Supponiamo che $f$ non sia identicamente nulla.
Da $P(0,0)$ si ottiene $f(f(0)^2)=0$, quindi esiste almeno un reale che annulla la funzione.
Se esiste un reale $t$ tale che $f(t)=0$ e $t≠1$, da $P(t,\frac{t}{t-1})$ si ottiene
$$f(0)=0$$

Da $P(x,0)$ si ottiene $f(x)=0$ assurdo $⇒f(t)=0$ se e solo se $t=1⇒f(0)^2=1⇒f(0)=−1$
Da $P(x,1)$ si ottiene
$$f(x+1)=f(x)+1$$
e induttivamente $f(x+n)=f(x)+n$.
Supponiamo per assurdo che esistano due reali $u,v$ distinti tali che $f(u)=f(v).$
Prendiamo $n$ tale che $(v+n)^2−4(u+n−1)≥0$, quindi esistono $x,y$ tali che $xy=u+n−1$ e $x+y=v+n,$ sostituendoli in $P$ si ottiene
$$f(f(x)f(y))+f(v+n)=f(u+n-1)\Rightarrow f(f(x)f(y))+f(v)+n=f(u)+n-1\Rightarrow f(f(x)f(y)+1)=0\Rightarrow f(x)f(y)=0$$
WLOG $f(y)=0⇒y=1⇒u+n−1=x=v+n−1⇒u=v$ assurdo $⇒f$ è iniettiva.
Da $P(x,−x)$ si ottiene
$$f(f(x)f(-x))=f(-x^2)+1=f(1-x^2)\Rightarrow f(x)f(-x)=1-x^2$$

Da $P(1−x,x)$ si ottiene
$$f(f(x)f(1-x))=f(x-x^2)\Rightarrow f(x)f(1-x)=x-x^2\Rightarrow x-x^2=f(x)+f(x)f(-x)=f(x)+1-x^2\Rightarrow f(x)=x-1$$

Da questo le tre soluzioni.
19.2 Lello01
Testo nascosto:
Sia $P(x,y)$ $:f(x+f(x+y) )+f(xy)=x+f(x+y)+yf(x)$
Le due soluzioni sono $f(x)=x$ e $f(x)=2-x$
Che verificano in quanto $x+x+y+xy=x+x+y+xy$
E $f(2-y)+2-xy=2-y+2y-xy$ che vale in quando $y+2-xy=2+y-xy.$
$P(0,y):f(f(y) )+f(0)=f(y)+yf(0)$
Ora ho due casi: o $f(0)=0$ o $f(0)≠0$
-se $f(0)=0$ allora $P(x,0)$ dà $f(x+f(x) )=x+f(x)$ $(2)$
$p(x-1,1)$ dà $f(x-1+f(x) )=x-1+f(x)$ $(3)$
Riscriviamo ora $P(x,y)$ come $f(x+f(x+y) )-x-yf(x)=f(x+y)-f(xy)$ da cui simmetrizzando
$$f(x+f(x+y) )-x-yf(x)=f(y+f(x+y))-y-xf(y)$$
Poniamo ora $y=-x$ da cui $f(x)-x+xf(x)=f(-x)+x-xf(-x)$ da cui sistemando viene
$(x+1)f(x)-2x=f(-x)(1-x)$ $∀x$ $(1)$
Se pongo $x=1$ in $(1)$ ottengo $f(1)=1$.
Definiamo ora $x+f(x)=A$
Per $(2)$ $f(A)=A$ mentre per $(3)$ $f(A-1)=A-1$
$P(1,A-1)$ ora dà $$f(1+A)+f(A-1)=1+A+A-1→f(1+A)=1+A$$
Quindi riprendendo la definizione di $A$ si ha che $f(x+f(x)+1)=x+f(x)+1$
Ora $P(x+1;-1)$ dà $$f(x+f(x)+1)+f(-x-1)=x+1+f(x)-f(x+1)→f(-x-1)=-f(x+1)$$
Da cui $∀x$ $f(x)=-f(-x)$
Ma allora sostituendo ciò in $(1)$ $$(x+1)f(x)-2x=f(x)(x-1)→2f(x)=2x→f(x)=x$$
-$f(0)≠0$
Allora $P(x-1,1)$ dà $f(x-1+f(x) )=x-1+f(x)$
Quindi $f$ ha almeno un punto fisso; consideriamo ora uno dei punti fissi che chiameremo $z$. Ma allora
$P(0,z):$ dà $$f(f(z) )+f(0)=f(z)+zf(0)→zf(0)=f(0)→z=1$$
Quindi l’unico punto fisso di $f(x)$ è $1$ ma in quanto $f(x-1+f(x) )=x-1+f(x)$ si deve avere che
$$x-1+f(x)=1→f(x)=2-x$$
19.3 Pit
Testo nascosto:
Sia $P(x,y)$ la prima disuguaglianza e $S(x,y)$ la seconda.
Da $P(1,a)$ si ottiene
$$af(1)≥a⇒f(1)≥1$$
Da $S(x,1)$ si ottiene induttivamente $f(n)≥n$.
Da $P\left(\frac{n}{m},m\right)$ si ottiene
$$f\left(\frac{n}{m}\right)f(m)\geq f(n)\geq n>0\Rightarrow f\left(\frac{n}{m}\right)>0$$

con $m,n$ interi positivi. Quindi per ogni razionale $x$ si ha $f(x)>0.$
Per ogni $x$ non intero, da $S(x−⌊x⌋,⌊x⌋)$ si ottiene
$$f(x)≥f(x−⌊x⌋)+f(⌊x⌋)≥f(⌊x⌋)≥⌊x⌋>x−1$$
Da $P(x,y)$ otteniamo induttivamente $f(x)^n≥f(x^n)$ e quindi
$$f(x)^n> x^n-1\Rightarrow f(x)>\sqrt[n]{x^n-1}$$

Per $x>1$ si ottiene
$$\lim\limits_{n\to\infty}\sqrt[n]{x^n-1}=e^{\lim\limits_{n\to\infty}\frac{\ln(x^n-1)}{n}}$$
dato che numeratore e denominatore del limite tendono entrambi ad infinito, per il teorema di de l'hopital si ottiene
$$\lim\limits_{n\to\infty}\sqrt[n]{x^n-1}=e^{\lim\limits_{n\to\infty}\frac{x^n\ln(x)}{x^n-1}}=e^{\ln(x)}=x\Rightarrow f(x)\geq x$$
per $x>1.$
Da $f(x)^n≥f(x^n)$ si ottiene
$$a^n=f(a)^n≥f(a^n)≥a^n⇒f(a^n)=a^n$$

Fissiamo $x$ maggiore di $1$ e prendiamo $n$ tale che $a^n>x+1$, da $S(a^n−x,x)$ otteniamo
$$a^n=f(a^n)=f((a^n-x)+x)\geq f(a^n-x)+f(x)\geq a^n-x+x=a^n$$
$⇒f(x)=x$ per ogni $x>1$
Da $S(x,y)$ otteniamo induttivamente
$$f(nx)≥nf(x)$$
Fissato $x$ e prendendo $n$ intero sufficientemente grande si ottiene
$$nx=f(nx)≥nf(x)⇒f(x)≤x$$
Da $P(x,n)$ si ottiene
$$f(x)f(n)≥f(nx)⇒nf(x)≥f(nx)=nx⇒f(x)≥x$$
E quindi $f(x)=x$ per ogni $x$ razionale, che è la tesi.
Ultima modifica di Pit il 19 mar 2020, 22:35, modificato 1 volta in totale.
Nessuno :?:
scambret
Messaggi: 734
Iscritto il: 23 mag 2012, 20:49
Località: Acquarica del Capo

Re: Algebra learning

Messaggio da scambret »

Sono contento di questo successo (inaspettato) - dovrei riprendere questo filone?
Avatar utente
Leonhard Euler
Messaggi: 42
Iscritto il: 01 gen 2018, 15:12

Re: Algebra learning

Messaggio da Leonhard Euler »

scambret ha scritto: 01 ott 2019, 22:02 Sono contento di questo successo (inaspettato) - dovrei riprendere questo filone?

Sicuramente la comunità te ne sarà più che riconoscente
« [...] ha cessato di calcolare e di vivere. » (Eulogia di Eulero)
Parmenide
Messaggi: 27
Iscritto il: 30 mag 2018, 21:24

Re: Algebra learning

Messaggio da Parmenide »

scambret ha scritto: 01 ott 2019, 22:02 Sono contento di questo successo (inaspettato) - dovrei riprendere questo filone?
Sarebbe fantastico!
Rispondi